Lippincott - Question of the Day

अब Quizwiz के साथ अपने होमवर्क और परीक्षाओं को एस करें!

Category: The Nursing Process A nurse is developing a nursing diagnosis for a client. Which information should she include? a) Actions to achieve goals b) Expected outcomes c) Factors influencing the client's problem d) Nursing history

???

Category: Mood, Adjustment, and Dementia Disorders A nurse is facilitating mandated group therapy for clients who have sexually abused children. Children who are victims of sexual abuse are typically: a) from any segment of the population. b) of low socioeconomic background. c) strangers to the abuser. d) willing to engage in sexual acts with adults.

???

Category: Basic Physical Care The nurse-manager of a home health facility includes which item in the capital budget? a) Salaries and benefits for her staff b) A $1,200 computer upgrade c) Office supplies d) Client-education materials costing $300

B) A $1,200 computer upgrade Explanation: Capital budgets generally include items valued at more than $500. Salaries and benefits are part of the personnel budget. Office supplies and client education materials are part of the operating budget.

Category: Psychotic Disorders A client with a tentative diagnosis of psychosis is admitted to the psychiatric unit. A physician orders the phenothiazine thioridazine 50 mg by mouth three times per day. Phenothiazines differ from central nervous system (CNS) depressants in their sedative effects by producing: A) deeper sleep than CNS depressants. B) greater sedation than CNS depressants. C) a calming effect from which the client is easily aroused. D) more prolonged sedative effects, making the client more difficult to arouse.

C) a calming effect from which the client is easily aroused. Explanation: Shortly after phenothiazine administration, a quieting and calming effect occurs, but the client is easily aroused, alert, and responsive and has good motor coordination.

Category: Medication and I.V. Administration The client is receiving an IV infusion of 5% dextrose in normal saline running at 125 ml/h. When hanging a new bag of fluid, the nurse notes swelling and hardness at the infusion site. The nurse should first: A) stop the flow of solution temporarily. B) apply a warm soak to the site. C) discontinue the infusion. D) irrigate the needle with normal saline.

C) discontinue the infusion. Reason: Signs of infiltration include slowing of the infusion and swelling, pain, hardness, pallor, and coolness of the skin at the site. If these signs occur, the IV line should be discontinued and restarted at another infusion site. The new anatomic site, time, and type of cannula used should be documented. The nurse may apply a warm soak to the site, but only after the IV line is discontinued. Parenteral administration of fluids should not be stopped intermittently. Stopping the flow does not treat the problem, nor does it address the client's needs for fluid replacement. Infiltrated IV sites should not be irrigated; doing so will only cause more swelling and pain.

Question Of The Day: Mood, Adjustment, and Dementia Disorders A nurse is evaluating a client's electrocardiogram (ECG). Which ECG change can result from amitriptyline (Elavil) therapy? A. Presence of U waves B. Depressed ST segment C. Widening QT interval D. Prolonged PR interval

C. Widening QT interval Reason: Amitriptyline therapy may cause a conduction delay, demonstrated by a widening QT interval on the ECG. U waves, a depressed ST segment, and a prolonged PR interval aren't typically induced by amitriptyline therapy.

Category: Toddler After teaching a group of parents about temper tantrums, the nurse knows the teaching has been effective when one of the parents states which of the following? a) "I will ignore the temper tantrum." b) "I should pick up the child during the tantrum." c) "I'll talk to my daughter during the tantrum." d) "I should put my child in time out."

a) "I will ignore the temper tantrum." Reason: Children who have temper tantrums should be ignored as long as they are safe. They should not receive either positive or negative reinforcement to avoid perpetuating the behavior. Temper tantrums are a toddler's way of achieving independence.

Category: Mood, Adjustment, and Dementia Disorders The wife of a 67-year-old client who has been taking imipramine (Tofranil) for 3 days asks the nurse why her husband isn't better. The nurse should tell the wife: a) "It takes 2 to 4 weeks before the full therapeutic effects are experienced." b) "Your husband may need an increase in dosage." c) "A different antidepressant may be necessary." d) "It can take 6 weeks to see if the medication will help your husband."

a) "It takes 2 to 4 weeks before the full therapeutic effects are experienced." Reason: Imipramine, a tricyclic antidepressant, typically requires 2 to 4 weeks of therapy before the full therapeutic effects are experienced. Because the client has been taking the drug for only 3 days, it is too soon to determine if the current dosage of imipramine is effective. It is also too soon to consider taking another antidepressant.

Category: Gastrointestinal Disorders A nurse preceptor is working with a student nurse who is administering medications. Which statement by the student indicates an understanding of the action of an antacid? a) "The action occurs in the stomach by increasing the pH of the stomach contents and decreasing pepsin activity." b) "The action occurs in the small intestine, where the drug coats the lining and prevents further ulceration." c) "The action occurs in the esophagus by increasing peristalsis and improving movement of food into the stomach." d) "The action occurs in the large intestine by increasing electrolyte absorption into the system that decreases pepsin absorption."

a) "The action occurs in the stomach by increasing the pH of the stomach contents and decreasing pepsin activity." Reason: The action of an antacid occurs in the stomach. The anions of an antacid combine with the acidic hydrogen cations secreted by the stomach to form water, thereby increasing the pH of the stomach contents. Increasing the pH and decreasing the pepsin activity provide symptomatic relief from peptic ulcer disease. Antacids don't work in the large or small intestine or in the esophagus.

Category: Endocrine and Metabolic Disorders A nurse is caring for a client with poorly managed diabetes mellitus who has a serious foot ulcer. When she informs him that the physician has ordered a wound care nurse to examine his foot, the client asks why he should see anyone other than this nurse. He states, "It's no big deal. I'll keep it covered and put antibiotic ointment on it." What is the nurse's best response? a) "We're very concerned about your foot and we want to provide the best possible care for you." b) "This is a big deal and you need to recognize how serious it is." c) "This is the physician's recommendation. The wound care nurse will see you today." d) "You could lose your foot if you don't see the wound care nurse."

a) "We're very concerned about your foot and we want to provide the best possible care for you." Reason: The client's response indicates that he's in denial and needs further insight and education about his condition. Letting the client know that the nurse has his best interests in mind helps him accept the wound-care nurse. Although telling the client that his condition is serious and that the wound care nurse will see him that day are true statements, they're much too direct and may increase client resistance. Telling the client he could lose his foot is inappropriate and isn't therapeutic communication.

Category: Medication and I.V. Administration A client is scheduled for an excretory urography at 10 a.m. An order directs the nurse to insert a saline lock I.V. device at 9:30 a.m.. The client requests a local anesthetic for the I.V. procedure and the physician orders lidocaine-prilocaine cream (EMLA cream). The nurse should apply the cream at: a) 7:30 a.m. b) 8:30 a.m. c) 9 a.m. d) 9:30 a.m.

a) 7:30 a.m. Reason: It takes up to 2 hours for lidocaine-prilocaine cream (EMLA cream) to anesthetize an insertion site. Therefore, if the insertion is scheduled for 9:30 a.m., EMLA cream should be applied at 7:30 a.m. The local anesthetic wouldn't be effective if the nurse administered it at the later times.

Category: Medication and I.V. Administration A client is scheduled for an excretory urography at 10 a.m. An order directs the nurse to insert a saline lock I.V. device at 9:30 a.m.. The client requests a local anesthetic for the I.V. procedure and the physician orders lidocaine-prilocaine cream (EMLA cream). The nurse should apply the cream at: a)7:30 a.m. b)8:30 a.m. c)9 a.m. d)9:30 a.m.

a) 7:30 a.m. Reason: It takes up to 2 hours for lidocaine-prilocaine cream (EMLA cream) to anesthetize an insertion site. Therefore, if the insertion is scheduled for 9:30 a.m., EMLA cream should be applied at 7:30 a.m. The local anesthetic wouldn't be effective if the nurse administered it at the later times.

Category: Neurosensory Disorders Which of the following statements would provide the best guide for activity during the rehabilitation period for a client who has been treated for retinal detachment? a) Activity is resumed gradually, and the client can resume her usual activities in 5 to 6 weeks. b) Activity level is determined by the client's tolerance; she can be as active as she wishes. c) Activity level will be restricted for several months, so she should plan on being sedentary. d) Activity level can return to normal and may include regular aerobic exercises.

a) Activity is resumed gradually, and the client can resume her usual activities in 5 to 6 weeks. Reason: The scarring of the retinal tear needs time to heal completely. Therefore, resumption of activity should be gradual; Successful healing should allow the client to return to her previous level of functioning

Category: Psychotic Disorders A newly admitted client diagnosed with paranoid schizophrenia is pacing rapidly and wringing his hands. He states that another client is out to get him. Then he says, "Protect me, select me, reject me." The nurse should next: a) Administer his oral PRN lorazepam (Ativan) and haloperidol (Haldol). b) Place the client in temporary seclusion before he has a chance to hurt others. c) Call the primary health care provider for a prescription for restraints. d) Ask the other clients to leave the immediate area.

a) Administer his oral PRN lorazepam (Ativan) and haloperidol (Haldol). Explanation: The client's anxiety as reflected in rapid pacing and clang associations is rising as a result of his paranoid delusions. Administering the Ativan and Haldol will help the anxiety and delusions. He is not threatening others at this point, so seclusion, restraints, and asking clients to leave the area is not necessary.

Category: Psychotic Disorders A newly admitted client diagnosed with paranoid schizophrenia is pacing rapidly and wringing his hands. He states that another client is out to get him. Then he says, "Protect me, select me, reject me." The nurse should next: a) Administer his oral PRN lorazepam (Ativan) and haloperidol (Haldol). b) Place the client in temporary seclusion before he has a chance to hurt others. c) Call the primary health care provider for a prescription for restraints. d) Ask the other clients to leave the immediate area.

a) Administer his oral PRN lorazepam (Ativan) and haloperidol (Haldol). Reason: The client's anxiety as reflected in rapid pacing and clang associations is rising as a result of his paranoid delusions. Administering the Ativan and Haldol will help the anxiety and delusions. He is not threatening others at this point, so seclusion, restraints, and asking clients to leave the area is not necessary.

Category: Intrapartum Period Which finding indicates placental detachment? a) An abrupt lengthening of the cord b) A decrease in the number of contractions c) Relaxation of the uterus d) Decreased vaginal bleeding

a) An abrupt lengthening of the cord Reason: An abrupt lengthening of the cord, an increase (not a decrease) in the number of contractions, and an increase (not a decrease) in vaginal bleeding are all indications that the placenta has detached from the wall of the uterus. Relaxation of the uterus isn't an indication for detachment of the placenta. b) A decrease in the number of contractions

Category: Foundations of Psychiatric Nursing A nurse makes a home visit to a client who was discharged from a psychiatric hospital. The client is irritable and walks about her room slowly and morosely. After 10 minutes, the nurse prepares to leave, but the client plucks at the nurse's sleeve and quickly asks for help rearranging her belongings. She also anxiously makes inconsequential remarks to keep the nurse with her. In view of the fact that the client has previously made a suicidal gesture, which of the following interventions by the nurse should be a priority at this time? a) Ask the client frankly if she has thoughts of or plans for committing suicide. b) Avoid bringing up the subject of suicide to prevent giving the client ideas of self-harm. c) Outline some alternative measures to suicide for the client to use during periods of sadness. d) To draw out the client, mention others the nurse has known who have felt like the client and attempted suicide.

a) Ask the client frankly if she has thoughts of or plans for committing suicide. Reason: Investigating the presence of suicidal thoughts and plans by overtly asking the client if she is thinking of or planning to commit suicide is a priority nursing action in this situation. Direct questioning about thoughts or plans related to self-harm does not give a person the idea to harm herself. Self-harm is an individual decision. Avoiding the subject when a client appears suicidal is unwise; the safest procedure is to investigate. It would be premature in this situation to outline alternative measures to suicide. Describing other clients who have attempted suicide is too indirect to be helpful and minimizes the client's feelings.

Category: Gastrointestinal Disorders A client with esophageal cancer decides against placement of a jejunostomy tube. Which ethical principle is a nurse upholding by supporting the client's decision? a) Autonomy b) Fidelity c) Nonmaleficence d) Veracity

a) Autonomy

Category: Gastrointestinal Disorders A client with esophageal cancer decides against placement of a jejunostomy tube. Which ethical principle is a nurse upholding by supporting the client's decision? a) Autonomy b) Fidelity c) Nonmaleficence d) Veracity

a) Autonomy Reason: Autonomy refers to an individual's right to make his own decisions. Fidelity is equated with faithfulness. Nonmaleficence is the duty to "do no harm." Veracity refers to telling the truth.

Category: The Neonate A neonate has a large amount of secretions. After vigorously suctioning the neonate, the nurse should assess for what possible result? a) Bradycardia. b) Rapid eye movement. c) Seizures. d) Tachycardia.

a) Bradycardia. Reason: due to potential vagus nerve stimulation from deep suctioning

Category: The Neonate A neonate has a large amount of secretions. After vigorously suctioning the neonate, the nurse should assess for what possible result? a) Bradycardia. b) Rapid eye movement. c) Seizures. d) Tachycardia.

a) Bradycardia. Reason: As a result of vigorous suctioning the nurse must watch for bradycardia due to potential vagus nerve stimulation. Rapid eye movement is not associated with vagus nerve stimulation. Vagal stimulation will not cause seizures or tachycardia.

Category: Anxiety Disorders Which of the following should the nurse teach a client with generalized anxiety disorder to help the client cope with anxiety? a) Cognitive and behavioral strategies. b) Issue avoidance and denial of problems. c) Rest and sleep. d) Withdrawal from role expectations and role relationships.

a) Cognitive and behavioral strategies. Reason: A client with generalized anxiety disorder needs to learn cognitive and behavioral strategies to cope with anxiety appropriately. In doing so, the client's anxiety decreases and becomes more manageable. The client may need assertiveness training, reframing, and relaxation exercises to adaptively deal with anxiety.

Category: Respiratory Disorders A nurse is caring for a client who has a tracheostomy and temperature of 103° F (39.4° C). Which intervention will most likely lower the client's arterial blood oxygen saturation? a) Endotracheal suctioning b) Encouragement of coughing c) Use of a cooling blanket d) Incentive spirometry

a) Endotracheal suctioning Reason: Endotracheal suctioning removes secretions as well as gases from the airway and lowers the arterial oxygen saturation (SaO2) level. Coughing and using an incentive spirometer improve oxygenation and should raise or maintain oxygen saturation. Because of superficial vasoconstriction, using a cooling blanket can lower peripheral oxygen saturation readings, but SaO2 levels wouldn't be affected.

Category: The Nursing Process The nurse is serving on the hospital ethics committee which is considering the ethics of a proposal for the nursing staff to search the room of a client diagnosed with substance abuse while he is off the unit and without his knowledge. Which of the following should be considered concerning the relationship of ethical and legal standards of behavior? a) Ethical standards are generally higher than those required by law. b) Ethical standards are equal to those required by law. c) Ethical standards bear no relationship to legal standards for behavior. d) Ethical standards are irrelevant when the health of a client is at risk.

a) Ethical standards are generally higher than those required by law. Reason: some behavior that is legally allowed might not be considered ethically appropriate. Legal and ethical standards are often linked, such as in the commandment "Thou shall not kill." Ethical standards are never irrelevant, though a client's safety the safety of others may pose an ethical dilemma for health care personnel. Searching a clients room when they are not there is a violation of privacy. Room searches can be done with a primary health care providers order and generally are done with the client present.

Category: Anxiety Disorders A client who recently developed paralysis of the arms is diagnosed with conversion disorder after tests fail to uncover a physical cause for the paralysis. Which intervention should the nurse include in the care plan for this client? a) Exercising the client's arms regularly b) Insisting that the client eat without assistance c) Working with the client rather than with the family d) Teaching the client how to use nonpharmacologic pain-control methods

a) Exercising the client's arms regularly Reason: To maintain the integrity of the affected areas and prevent muscle wasting and contractures, the nurse should help the client perform regular passive range-of-motion exercises with his arms. The nurse shouldn't insist that the client use his arms to perform such functions as eating without assistance, because he can't consciously control his symptoms and move his arms; such insistence may anger the client and endanger the therapeutic relationship. The nurse should include family members in the client's care because they may be contributing to the client's stress or conflict and are essential to helping him regain function of his arms. The client isn't experiencing pain and, therefore, doesn't need education regarding pain management.

Category: Postpartum Period The nurse is assessing a client at her postpartum checkup 6 weeks after a vaginal delivery. The mother is bottle feeding her baby. Which client finding indicates a problem at this time? a) Firm fundus at the symphysis. b) White, thick vaginal discharge. c) Striae that are silver in color. d) Soft breasts without milk.

a) Firm fundus at the symphysis. Reason: by 4 to 6 weeks postpartum, the fundus should be deep in the pelvis and the size of a non-pregnant uterus. Subinvolution, caused by infection or retained placental fragments, is a problem associated with a uterus that is larger than expected at this time. Normal expectations include a white, thick vaginal discharge, striae that are beginning to fade to silver, and breasts that are soft without evidence of milk production (in a bottle feeding mother).

Category: Intrapartum Period Which nursing action is required before a client in labor receives epidural anesthesia? a) Give a fluid bolus of 500 ml. b) Check for maternal pupil dilation. c) Assess maternal reflexes. d) Assess maternal gait.

a) Give a fluid bolus of 500 ml. Reason: One of the major adverse effects of epidural administration is hypotension. Therefore, a 500-ml fluid bolus is usually administered to prevent hypotension in the client who wishes to receive an epidural for pain relief. Assessing maternal reflexes, pupil response, and gait isn't necessary.

Category: Intrapartum Period Which nursing action is required before a client in labor receives epidural anesthesia? a)Give a fluid bolus of 500 ml. b)Check for maternal pupil dilation. c)Assess maternal reflexes. d)Assess maternal gait.

a) Give a fluid bolus of 500 ml. Reason: One of the major adverse effects of epidural administration is hypotension. Therefore, a 500-ml fluid bolus is usually administered to prevent hypotension in the client who wishes to receive an epidural for pain relief. Assessing maternal reflexes, pupil response, and gait isn't necessary.

Category: Toddler When performing a physical assessment on an 18-month-old child, which of the following would be best? a) Have a parent hold the toddler. b) Assess the ears and mouth first. c) Carry out the assessment from head to toe. d) Assess motor function by having the child run and walk.

a) Have a parent hold the toddler.

Category: Musculoskeletal Disorders A nurse is caring for a client with lower back pain who is scheduled for myelography using metrizamide (a water-soluble contrast dye). After the test, the nurse should place the client in which position? a) Head of the bed elevated 45 degrees b) Prone c) Supine with feet raised d) Supine with the head lower than the trunk

a) Head of the bed elevated 45 degrees Reason: After a myelogram, positioning depends on the dye injected. When a water-soluble dye such as metrizamide is injected, the head of the bed is elevated to a 45-degree angle to slow the upward dispersion of the dye. The prone and supine positions are contraindicated when a water-soluble contrast dye is used. The client should be positioned supine with the head lower than the trunk after an air-contrast study. *A myelogram is a diagnostic imaging procedure done by a radiologist. It uses a contrast dye and X-rays or computed tomography (CT) to look for problems in the spinal canal, including the spinal cord, nerve roots, and other tissues. It is also known as myelography.

Category: School-age Child On initial assessment of a 7-year-old child with rheumatic fever, which of the following would require contacting the primary care provider immediately? a) Heart rate of 150 beats/minute. b) Swollen and painful knee joints. c) Twitching in the extremities. d) Red rash on the trunk.

a) Heart rate of 150 beats/minute. Reason: A heart rate of 150 beats/minute is very high for a 7-year-old child and may indicate carditis. For this age group, the normal heart rate while awake is 70 to 110 beats/minute. Swollen and painful joints such as the knee are characteristic findings in the child with rheumatic fever and do not require immediate physician notification. Twitching in the extremities, known as chorea, is a characteristic finding in a child with rheumatic fever and does not require immediate physician notification. A red rash on the trunk typically indicates rheumatic fever and does not require immediate physician notification.

Category: School-age Child A parent asks the nurse about head lice (pediculosis capitis) infestation during a visit to the clinic. Which of the following symptoms should the nurse tell the parent is most common in a child infected with head lice? a) Itching of the scalp. b) Scaling of the scalp. c) Serous weeping on the scalp surface. d) Pinpoint hemorrhagic spots on the scalp surface.

a) Itching of the scalp.

Category: Gastrointestinal Disorders The comatose victim of the car accident is to have a gastric lavage. Which of the following positions would be most appropriate for the client during this procedure? a) Lateral. b) Supine. c) Trendelenburg's. d) Lithotomy.

a) Lateral Reason: An unconscious client is best positioned in a lateral or semiprone position because these positions allow the jaw and tongue to fall forward, fascilitate the drainage of secretions, and prevent aspiration.

Category: Endocrine and Metabolic Disorders A 75-year-old client is newly diagnosed with diabetes. The nurse is instructing him about blood glucose testing. After the session, the client states, "I can't be expected to remember all this stuff." The nurse should recognize this response as most likely related to which of the following? a) Moderate to severe anxiety. b) Disinterest in the illness. c) Early-onset dementia. d) Normal reaction to learning a new skill.

a) Moderate to severe anxiety. Reason: Anxiety, especially at higher levels, interferes with learning and memory retention . After the client's anxiety lessens, it will be easier for him to learn the steps of the blood glucose monitoring. Because the client's illness is a chronic, lifelong illness that severely changes his lifestyle, it is unlikely that he is uninterested in the illness or how to treat it. It is also unlikely that dementia would be the cause of the client'sfrustration and lack of memory. The client's response indicates anxiety. Client responses that would indicate lessening anxiety would be questions to the nurse or requests to repeat part of the instruction. CN: Psychosocial

Category: Endocrine and Metabolic Disorders A 75-year-old client is newly diagnosed with diabetes. The nurse is instructing him about blood glucose testing. After the session, the client states, "I can't be expected to remember all this stuff." The nurse should recognize this response as most likely related to which of the following? a) Moderate to severe anxiety. b) Disinterest in the illness. c) Early-onset dementia. d) Normal reaction to learning a new skill.

a) Moderate to severe anxiety. Reason: Anxiety, especially at higher levels, interferes with learning and memory retention. After the client's anxiety lessens, it will be easier for him to learn the steps of the blood glucose monitoring. Because the client's illness is a chronic, lifelong illness that severely changes his lifestyle, it is unlikely that he is uninterested in the illness or how to treat it. It is also unlikely that dementia would be the cause of the client's frustration and lack of memory. The client's response indicates anxiety. Client responses that would indicate lessening anxiety would be questions to the nurse or requests to repeat part of the instruction.

Category: Infant Which of the following should the nurse use to determine achievement of the expected outcome for an infant with severe diarrhea and a nursing diagnosis of Deficient fluid volume related to passage of profuse amounts of watery diarrhea? a) Moist mucous membranes. b) Passage of a soft, formed stool. c) Absence of diarrhea for a 4-hour period. d) Ability to tolerate intravenous fluids well.

a) Moist mucous membranes. Reason: The outcome of moist mucous membranes indicates adequate hydration and fluid balance, showing that the problem of fluid volume deficit has been corrected. Although a normal bowel movement, ability to tolerate intravenous fluids, and an increasing time interval between bowel movements are all positive signs, they do not specifically address the problem of deficient fluid volume.

Category: Infant Which of the following should the nurse use to determine achievement of the expected outcome for an infant with severe diarrhea and a nursing diagnosis of Deficient fluid volume related to passage of profuse amounts of watery diarrhea? a) Moist mucous membranes. b) Passage of a soft, formed stool. c) Absence of diarrhea for a 4-hour period. d) Ability to tolerate intravenous fluids well.

a) Moist mucous membranes. Reason: The outcome of moist mucous membranes indicates adequate hydration and fluid balance, showing that the problem of fluid volume deficit has been corrected. Although a normal bowel movement, ability to tolerate intravenous fluids, and an increasing time interval between bowel movements are all positive signs, they do not specifically address the problem of deficient fluid volume.

Category: Gastrointestinal Disorders Prochlorperazine (Compazine) is prescribed postoperatively. The nurse should evaluate the drug's therapeutic effect when the client expresses relief from which of the following? a) Nausea. b) Dizziness. c) Abdominal spasms. d) Abdominal distention.

a) Nausea. Reason: Prochlorperazine is administered postoperatively to control nausea and vomiting. Prochlorperazine is also used in psychotherapy because of its effects on mood and behavior. It is not used to treat dizziness, abdominal spasms, or abdominal distention.

Category: Gastrointestinal Disorders Prochlorperazine (Compazine) is prescribed postoperatively. The nurse should evaluate the drug's therapeutic effect when the client expresses relief from which of the following? a)Nausea. b)Dizziness. c)Abdominal spasms. d)Abdominal distention.

a) Nausea. Reason: Prochlorperazine is administered postoperatively to control nausea and vomiting. Prochlorperazine is also used in psychotherapy because of its effects on mood and behavior. It is not used to treat dizziness, abdominal spasms, or abdominal distention.

Category: Postpartum Period While assessing the fundus of a multiparous client on the first postpartum day, the nurse performs handwashing and dons clean gloves. Which of the following should the nurse do next? a) Place the nondominant hand above the symphysis pubis and the dominant hand at the umbilicus. b) Ask the client to assume a side-lying position with the knees flexed. c) Perform massage vigorously at the level of the umbilicus if the fundus feels boggy. d) Place the client on a bedpan in case the uterine palpation stimulates the client to void.

a) Place the nondominant hand above the symphysis pubis and the dominant hand at the umbilicus. Reason: The nurse should place the nondominant hand above the symphysis pubis and the dominant hand at the umbilicus to palpate the fundus. This prevents uterine inversion and trauma, which can be very painful to the client. The nurse should ask the client to assume a supine, not side-lying, position with the knees flexed. The fundus can be palpated in this position and the perineal pads can be evaluated for lochia amounts. The fundus should be massaged gently if the fundus feels boggy. Vigorous massaging may fatigue the uterus and cause it to become firm and then boggy again. The nurse should ask the client to void before fundal evaluation. A full bladder can cause discomfort to the client, the uterus to be deviated to one side, and postpartum hemorrhage.

Category: Basic Physical Care A nurse takes informed consent from a client scheduled for abdominal surgery. Which of the following is the most appropriate principle behind informed consent? a) Protects the client's right to self-determination in health care decision making. b) Helps the client refuse treatment that he or she does not wish to undergo. c) Helps the client to make a living will regarding future health care required. d) Provides the client with in-depth knowledge about the treatment options available.

a) Protects the client's right to self-determination in health care decision making. Reason: Informed consent protects the client's right to self-determination in health care decision making. Informed consent helps the client to refuse a treatment that the client does not wish to undergo and helps the client to gain in-depth knowledge about the treatment options available, but the most important function is to encourage shared decision making. Informed consent does not help the client to make a living will.

Category: Basic Physical Care A nurse takes informed consent from a client scheduled for abdominal surgery. Which of the following is the most appropriate principle behind informed consent? a)Protects the client's right to self-determination in health care decision making. b)Helps the client refuse treatment that he or she does not wish to undergo. c)Helps the client to make a living will regarding future health care required. d)Provides the client with in-depth knowledge about the treatment options available.

a) Protects the client's right to self-determination in health care decision making. Reason: Informed consent protects the client's right to self-determination in health care decision making. Informed consent helps the client to refuse a treatment that the client does not wish to undergo and helps the client to gain in-depth knowledge about the treatment options available, but the most important function is to encourage shared decision making. Informed consent does not help the client to make a living will.

Category: The Nursing Process A client has the following arterial blood gas values: pH, 7.30; PaO2, 89 mm Hg; PaCO2, 50 mm Hg; and HCO3-, 26 mEq/L. Based on these values, the nurse should suspect which condition? a) Respiratory acidosis b) Respiratory alkalosis c) Metabolic acidosis d) Metabolic alkalosis

a) Respiratory acidosis Explanation: This client has a below-normal (acidic) blood pH value and an above-normal partial pressure of arterial carbon dioxide (PaCO2) value, indicating respiratory acidosis. In respiratory alkalosis, the pH value is above normal and the PaCO2 value is below normal. In metabolic acidosis, the pH and bicarbonate (HCO3-) values are below normal. In metabolic alkalosis, the pH and HCO3- values are above normal.

Category: The Neonate Just after delivery, a nurse measures a neonate's axillary temperature at 94.1° F (34.5° C). What should the nurse do? a) Rewarm the neonate gradually. b) Rewarm the neonate rapidly. c) Observe the neonate hourly. d) Notify the physician when the neonate's temperature is normal.

a) Rewarm the neonate gradually.

Category: Toddler A nurse should expect a 3-year-old child to be able to perform which action? a) Ride a tricycle b) Tie his shoelaces c) Roller-skate d) Jump rope

a) Ride a tricycle Reason: The nurse should expect the child to ride a tricycle because, at age 3, gross motor development and refinement in eye-hand coordination enable a child to perform such an action. The fine motor skills required to tie shoelaces and the gross motor skills required for roller-skating and jumping rope develop around age 5.

Category: Neurosensory Disorders A nurse is monitoring a client for adverse reactions to atropine (Atropine Care) eyedrops. Systemic absorption of atropine sulfate through the conjunctiva can cause which adverse reaction? a) Tachycardia b) Increased salivation c) Hypotension d) Apnea

a) Tachycardia Reason: Systemic absorption of atropine can cause tachycardia, palpitations, flushing, dry skin, ataxia, and confusion. To minimize systemic absorption, the client should apply digital pressure over the punctum at the inner canthus for 2 to 3 minutes after instilling the drops. The drug also may cause dry mouth. It isn't known to cause hypotension or apnea.

Category: Gastrointestinal Disorders A client with a bleeding ulcer is vomiting bright red blood. The nurse should assess the client for which of the following indicators of early shock? a) Tachycardia. b) Dry, flushed skin. c) Increased urine output. d) Loss of consciousness.

a) Tachycardia. Reason: In early shock, the body attempts to meet its perfusion needs through tachycardia, vasoconstriction, and fluid conservation. The skin becomes cool and clammy. Urine output in early shock may be normal or slightly decreased. The client may experience increased restlessness and anxiety from hypoxia, but loss of consciousness is a late sign of shock.

Category: Anxiety Disorders A client with obsessive-compulsive disorder may use reaction formation as a defense mechanism to cope with anxiety and stress. What typically occurs in reaction formation? a) The client assumes an attitude that contradicts an impulse he harbors. b) The client believes his thoughts can control other people and events. c) The client persistently thinks and talks about a particular idea or subject. d) The client uses a specific act to negate a previous act.

a) The client assumes an attitude that contradicts an impulse he harbors.

Category: Foundations of Psychiatric Nursing A client is irritable and hostile. He becomes agitated and verbally lashes out when his personal needs are not immediately met by the staff. When the client's request for a pass is refused by the primary care provider, he utters a stream of profanities. Which of the following statements best describes the client's behavior? a) The client's anger is not intended personally. b) The client's anger is a reliable sign of serious pathology. c) The client's anger is an intended attack on the primary care provider's skills d) The client's anger is a sign that his condition is improving.

a) The client's anger is not intended personally. Explanation: Staff members sometimes are the recipients of a client's angry behavior because they are safe targets and are available for attack. The display of anger is rarely intended to be personal. Such behavior is not necessarily a sign of serious pathology but must be weighed in conjunction with other behaviors. An angry outburst is not an attack on a primary care provider's skills. While not necessarily pathologic, the client's behavior isn't a sign that his condition is improving.

Category: The Nursing Process A nurse has received change-of-shift-report and is briefly reviewing the documentation about a client in the client's medical record. A recent entry reads, "Client was upset throughout the morning." How could the charting entry be best improved? a) The entry should include clearer descriptions of the client's mood and behavior. b) The entry should avoid mentioning cognitive or psychosocial issues. c) The entry should list the specific reasons that the client was upset. d) The entry should specify the subsequent interventions that were performed.

a) The entry should include clearer descriptions of the client's mood and behavior. Reason: Entries in the medical record should be precise, descriptive, and objective. An adjective such as "upset" is unclear and open to many interpretations. As such, the nurse should elaborate on this description so a reader has a clearer understanding of the client's state of mind. Stating the apparent reasons that the client was "upset" does not resolve the ambiguity of this descriptor. Cognitive and psychosocial issues are valid components of the medical record. Responses and interventions should normally follow assessment data but the data themselves must first be recorded accurately.

Category: Oncologic Disorders A nurse is caring for a client receiving chemotherapy. Which nursing action is most appropriate for handling chemotherapeutic agents? a) Wear disposable gloves and protective clothing. b) Break needles after the infusion is discontinued. c) Disconnect I.V. tubing with gloved hands. d) Throw I.V. tubing in the trash after the infusion is stopped.

a) Wear disposable gloves and protective clothing. Explanation: A nurse must wear disposable gloves and protective clothing to prevent skin contact with chemotherapeutic agents. The nurse shouldn't recap or break needles. The nurse should use a sterile gauze pad when priming I.V. tubing, connecting and disconnecting tubing, inserting syringes into vials, breaking glass ampules, or other procedures in which chemotherapeutic agents are being handled. Contaminated needles, syringes, I.V. tubes, and other equipment must be disposed of in a leak-proof, puncture-resistant container

Category: Endocrine and Metabolic Disorders A client with type 1 diabetes must undergo bowel resection in the morning. How should the nurse proceed while caring for him on the morning of surgery? a) Administer half of the client's typical morning insulin dose as ordered. b) Administer an oral antidiabetic agent as ordered. c) Administer an I.V. insulin infusion as ordered. d) Administer the client's normal daily dose of insulin as ordered.

a) administer half of the client's typical morning insulin dose as ordered. Reason: If the nurse administers the client's normal daily dose of insulin while he's nothing-by-mouth status before surgery, he'll experience hypoglycemia. Therefore, the nurse should administer half the daily insulin dose as ordered.

Category: Mood, Adjustment, and Dementia Disorders Family members of a client with bipolar disorder tell a nurse that they are concerned that the client is becoming manic. The nurse knows that the manic phase is marked by: a) flight of ideas and inflated self-esteem. b) increased sleep and greater distractibility. c) decreased self-esteem and increased physical restlessness. d) obsession with following rules and maintaining order.

a) flight of ideas and inflated self-esteem. Explanation: The manic phase of bipolar disorder is characterized by recurrent episodes of a persistently euphoric and expansive or irritable mood. This phase is diagnosed if the client experiences four of the following signs and symptoms for at least 1 week: flight of ideas; inflated self-esteem; unusual talkativeness; increased social, occupational, or sexual activity; physical restlessness; a decreased need for sleep; increased distractibility; and excessive involvement in activities with a high potential for painful but unrecognized consequences. Obsession with following rules and maintaining order characterizes obsessive-compulsive disorder.

Category: Neurosensory Disorders A nurse on a rehabilitation unit is caring for a client who sustained a head injury in a motor vehicle accident. She notes that the client has become restless and agitated during therapy; previous documentation described the client as cooperative during therapy sessions. The nurse's priority action should be to: a) gather assessment data and notify the physician of the change in the client's status. b) ask the physician to order an antipsychotic medication for the client. c) consult with the social worker about the possibility of discharging the client from the facility. d) tell the client that she'll punish him if he doesn't behave.

a) gather assessment data and notify the physician of the change in the client's status. Explanation: A client with a head injury who experiences a change in cognition requires further assessment and evaluation, and the nurse should notify the physician of the change in the client's status. The physician should rule out all possible medical causes of the change in mental status before ordering antipsychotic medications or considering discharging the client from the facility. A nurse shouldn't threaten a client with punishment; doing so is a violation of the client's rights.

Category: Infant A nurse is caring for a family whose infant has anencephaly. The most appropriate nursing intervention is to: a) help the family prepare for the infant's imminent death. b) implement measures to facilitate the attachment process. c) provide emotional support so the family can adjust to the birth of an infant with health problems. d) prepare the family for the extensive surgical procedures the infant will require.

a) help the family prepare for the infant's imminent death.

Category: Cardiovascular Disorders A client with chronic heart failure is receiving digoxin (Lanoxin), 0.25 mg by mouth daily, and furosemide (Lasix), 20 mg by mouth twice daily. The nurse instructs the client to notify the physician if nausea, vomiting, diarrhea, or abdominal cramps occur because these signs and symptoms may signal digoxin toxicity. Digoxin toxicity may also cause: a) visual disturbances. b) taste and smell alterations. c) dry mouth and urine retention.

a) visual disturbances.

Category: Cardiovascular Disorders A client with chronic heart failure is receiving digoxin (Lanoxin), 0.25 mg by mouth daily, and furosemide (Lasix), 20 mg by mouth twice daily. The nurse instructs the client to notify the physician if nausea, vomiting, diarrhea, or abdominal cramps occur because these signs and symptoms may signal digoxin toxicity. Digoxin toxicity may also cause: a)visual disturbances. b)taste and smell alterations. c)dry mouth and urine retention. d)nocturia and sleep disturbances.

a) visual disturbances. Reason: Digoxin toxicity may cause visual disturbances (such as, flickering flashes of light, colored or halo vision, photophobia, blurring, diplopia, and scotomata), central nervous system abnormalities (such as headache, fatigue, lethargy, depression, irritability and, if profound, seizures, delusions, hallucinations, and memory loss), and cardiovascular abnormalities (abnormal heart rate and arrhythmias). Digoxin toxicity doesn't cause taste and smell alterations. Dry mouth and urine retention typically occur with anticholinergic agents, not inotropic agents such as digoxin. Nocturia and sleep disturbances are adverse effects of furosemide — especially if the client takes the second daily dose in the evening, which may cause diuresis at night.

Category: Gastrointestinal Disorders Which of the following client statements indicates that the client with hepatitis B understands discharge teaching? a) "I will not drink alcohol for at least 1 year." b) "I must avoid sexual intercourse." c) "I should be able to resume normal activity in a week or two. d) "Because hepatitis B is a chronic disease, I know I will always be jaundiced."

a)"I will not drink alcohol for at least 1 year." Reason: It is important that the client understand that alcohol should be avoided for at least 1 year after an episode of hepatitis. Sexual intercourse does not need to be avoided, but the client should be instructed to use condoms until the hepatitis B surface antigen measurement is negative. The client will need to restrict activity until liver function test results are normal; this will not occur within 1 to 2 weeks. Jaundice will subside as the client recovers; it is not a permanent condition.

Category: Antepartum Period A pregnant client in her third trimester is started on chlorpromazine (Thorazine) 25 mg four times daily. Which of the following instructions is most important for the nurse to include in the client's teaching plan? a) "Don't drive because there's a possibility of seizures occurring." b) "Avoid going out in the sun without a sunscreen with a sun protection factor of 25." c) "Stop the medication immediately if constipation occurs." d) "Tell your doctor if you experience an increase in blood pressure."

b) "Avoid going out in the sun without a sunscreen with a sun protection factor of 25." Reason: Chlorpromazine is a low-potency antipsychotic that is likely to cause sun-sensitive skin. Therefore the client needs instructions about using sunscreen with a sun protection factor of 25 or higher. Typically, chlorpromazine is not associated with an increased risk of seizures. Although constipation is a common adverse effect of this drug, it can be managed with diet, fluids, and exercise. The drug does not need to be discontinued. Chlorpromazine is associated with postural hypotension, not hypertension. Additionally, if postural hypotension occurs, safety measures, such as changing positions slowly and dangling the feet before arising, not stopping the drug, are instituted.

Category: Cardiovascular Disorders Before discharge, which instruction should a nurse give to a client receiving digoxin (Lanoxin)? a) "Take an extra dose of digoxin if you miss one dose." b) "Call the physician if your heart rate is above 90 beats/minute." c) "Call the physician if your pulse drops below 80 beats/minute." d) "Take digoxin with meals."

b) "Call the physician if your heart rate is above 90 beats/minute."

Category: The Nursing Process A client complains of severe abdominal pain. To elicit as much information as possible about the pain, the nurse should ask: a) "Do you have the pain all the time?" b) "Can you describe the pain?" c) "Where does it hurt the most?" d) "Is the pain stabbing like a knife?"

b) "Can you describe the pain?" Reason: Asking an open-ended question such as "Can you describe the pain?" encourages the client to describe any and all aspects of the pain in his own words. The other options are likely to elicit less information because they're more specific and would limit the client's response.

Category: Infant A 10-month-old child with recurrent otitis media is brought to the clinic for evaluation. To help determine the cause of the child's condition, the nurse should ask the parents: a) "Does water ever get into the baby's ears during shampooing?" b) "Do you give the baby a bottle to take to bed?" c) "Have you noticed a lot of wax in the baby's ears?" d) "Can the baby combine two words when speaking?"

b) "Do you give the baby a bottle to take to bed?" Reason: In a young child, the eustachian tube is relatively short, wide, and horizontal, promoting drainage of secretions from the nasopharynx into the middle ear. Therefore, asking if the child takes a bottle to bed is appropriate because drinking while lying down may cause fluids to pool in the pharyngeal cavity, increasing the risk of otitis media. Asking if the parent noticed earwax, or cerumen, in the external ear canal is incorrect because wax doesn't promote the development of otitis media. During shampooing, water may become trapped in the external ear canal by large amounts of cerumen, possibly causing otitis external (external ear inflammation) as opposed to internal ear inflammation. Asking if the infant can combine two words is incorrect because a 10-month-old child isn't expected to do so.

Category: Gastrointestinal Disorders A home health nurse who sees a client with diverticulitis is evaluating teaching about dietary modifications necessary to prevent future episodes. Which statement by the client indicates effective teaching? a) "I'll increase my intake of protein during exacerbations." b) "I should increase my intake of fresh fruits and vegetables during remissions." c) "I'll snack on nuts, olives, and popcorn during flare-ups." d) "I'll incorporate foods rich in omega-3 fatty acids into my diet."

b) "I should increase my intake of fresh fruits and vegetables during remissions."

Category: Antepartum Period The nurse has discussed sexuality issues during the prenatal period with a primigravida who is at 32 weeks' gestation. She has had one episode of preterm labor. The nurse determines that the client understands the instructions when she says: a) "I can resume sexual intercourse when the bleeding stops." b) "I should not get sexually aroused or have any nipple stimulation." c) "I can resume sexual intercourse in 1 to 2 weeks." d) "I should not have sexual intercourse until my next prenatal visit."

b) "I should not get sexually aroused or have any nipple stimulation." Reason: This client has already had one episode of preterm labor at 32 weeks' gestation. Sexual intercourse, arousal, and nipple stimulation may result in the release of oxytocin which can contribute to continued preterm labor and early delivery. The client should be advised to refrain from these activities until closer to term, which is 6 to 8 weeks later. Telling the client that intercourse is acceptable after the bleeding stops is incorrect and may lead to early delivery of a preterm neonate. The client should not have intercourse for at least 6 weeks because of the danger of inducing labor. There is no indication when the client's next prenatal visit is scheduled.

Category: Adolescent A 17-year-old client who has been taking an antidepressant for six weeks has returned to the clinic for a medication check. When the nurse talks with the client and her mother, the mother reports that she has to remind the client to take her antidepressant every day. The client says, "Yeah, I'm pretty bad about remembering to take my meds, but I never miss a dose because Mom always bugs me about taking it." Which of the following responses would be effective for the nurse to make to the client? a) "It's a good thing your mom takes care of you by reminding you to take your meds."b b) "It seems there are some difficulties with being responsible for your medications that we need to address". c) "You'll never be able to handle your medication administration at college next year if you're so dependent on her." d) "I'm surprised your mother allows you to be so irresponsible."

b) "It seems there are some difficulties with being responsible for your medications that we need to address". Reason: The client and mother need to address the issue of responsibility for medication administration. Reinforcing the mother'soverinvolvement in medication taking or making negative comments about the client and mother t are unlikely to engage them in problem solving about the matter.

Category: Adolescent A 17-year-old client who has been taking an antidepressant for six weeks has returned to the clinic for a medication check. When the nurse talks with the client and her mother, the mother reports that she has to remind the client to take her antidepressant every day. The client says, "Yeah, I'm pretty bad about remembering to take my meds, but I never miss a dose because Mom always bugs me about taking it." Which of the following responses would be effective for the nurse to make to the client? a) "It's a good thing your mom takes care of you by reminding you to take your meds." b) "It seems there are some difficulties with being responsible for your medications that we need to address". c) "You'll never be able to handle your medication administration at college next year if you're so dependent on her." d) "I'm surprised your mother allows you to be so irresponsible."

b) "It seems there are some difficulties with being responsible for your medications that we need to address". Reason: The client and mother need to address the issue of responsibility for medication administration and only Option 2 opens that subject to discussion. Option 1 reinforces the mother's over-involvement in medication taking. Options 3 and 4 make negative comments about the client and mother that are unlikely to engage them in problem-solving about the matter.

Category: Psychotic Disorders The mother of a client with chronic undifferentiated schizophrenia calls the visiting nurse in the outpatient clinic to report that her daughter has not answered the phone in 10 days. "She was doing so well for months. I don't know what's wrong. I'm worried." Which of the following responses by the nurse is most appropriate? a) "Maybe she's just mad at you. Did you have an argument?" b) "She may have stopped taking her medications. I'll check on her." c) "Don't worry about this. It happens sometimes." d) "Go over to her apartment and see what's going on."

b) "She may have stopped taking her medications. I'll check on her." Reason: Noncompliance with medications is common in the client with chronic undifferentiated schizophrenia. The nurse has the responsibility to assess this situation. Asking the mother if they've argued or if the client is mad at the mother or telling the mother to go over to the apartment and see what's going on places the blame and responsibility on the mother and therefore is inappropriate. Telling the mother not to worry ignores the seriousness of the client's symptoms.

Category: Immune and Hematologic Disorders A client with pernicious anemia asks why she must take vitamin B12 injections for the rest of her life. Which is the nurse's best response? a) "The reason for your vitamin deficiency is an inability to absorb the vitamin because the stomach is not producing sufficient acid." b) "The reason for your vitamin deficiency is an inability to absorb the vitamin because the stomach is not producing sufficient intrinsic factor." c) "The reason for your vitamin deficiency is an excessive excretion of the vitamin because of kidney dysfunction." d) "The reason for your vitamin deficiency is an increased requirement for the vitamin because of rapid red blood cell production."

b) "The reason for your vitamin deficiency is an inability to absorb the vitamin because the stomach is not producing sufficient intrinsic factor." Reason: Most clients with pernicious anemia have deficient production of intrinsic factor in the stomach. Intrinsic factor attaches to the vitamin in the stomach and forms a complex that allows the vitamin to be absorbed in the small intestine. The stomach is producing enough acid, there is not an excessive excretion of the vitamin, and there is not a rapid production of RBCs in this condition.

Category: Immune and Hematologic Disorders A client with pernicious anemia asks why she must take vitamin B12 injections for the rest of her life. Which is the nurse's best response? a)"The reason for your vitamin deficiency is an inability to absorb the vitamin because the stomach is not producing sufficient acid." b)"The reason for your vitamin deficiency is an inability to absorb the vitamin because the stomach is not producing sufficient intrinsic factor." c)"The reason for your vitamin deficiency is an excessive excretion of the vitamin because of kidney dysfunction." d)"The reason for your vitamin deficiency is an increased requirement for the vitamin because of rapid red blood cell production."

b) "The reason for your vitamin deficiency is an inability to absorb the vitamin because the stomach is not producing sufficient intrinsic factor." Reason: Most clients with pernicious anemia have deficient production of intrinsic factor in the stomach. Intrinsic factor attaches to the vitamin in the stomach and forms a complex that allows the vitamin to be absorbed in the small intestine. The stomach is producing enough acid, there is not an excessive excretion of the vitamin, and there is not a rapid production of red blood cells in this condition.

Category: Foundations of Psychiatric Nursing A nurse is instructing a client with bipolar disorder on proper use of lithium carbonate (Eskalith), the drug's adverse effects, and symptoms of lithium toxicity. Which client statement indicates that additional teaching is required? a) "I can still eat my favorite salty foods." b) "When my moods fluctuate, I'll increase my dose of lithium." c) "A good blood level of the drug means the drug concentration has stabilized." d) "Eating too much watermelon will affect my lithium level."

b) "When my moods fluctuate, I'll increase my dose of lithium." Reason: A client who states that he'll increase his dose of lithium if his mood fluctuates requires additional teaching because increasing the dose of lithium without evaluating the client's laboratory values can cause serious health problems, such as lithium toxicity, overdose, and renal failure. Clients taking lithium don't need to limit their sodium intake. A low-sodium diet causes lithium retention. A therapeutic lithium blood level indicates that the drug concentration has stabilized. The client demonstrates effective teaching by stating his lithium levels will be affected by foods that have a diuretic effect, such as watermelon, cantaloupe, grapefruit juice, and cranberry juice.

Category: Antepartum Period A nurse is using Doppler ultrasound to assess a pregnant woman. When should the nurse expect to hear fetal heart tones? a) 7 weeks' gestation b) 11 weeks' gestation c) 17 weeks' gestation d) 21 weeks' gestation

b) 11 weeks Reason: Using Doppler ultrasound, fetal heart tones may be heard as early as the 11th week of pregnancy. Using a stethoscope, fetal heart tones may be heard between 17 and 20 weeks of gestation.

Category: Endocrine and Metabolic Disorders A nurse is teaching a client with diabetes mellitus about self-management of his condition. The nurse should instruct the client to administer 1 unit of insulin for every: a) 10 g of carbohydrates. b) 15 g of carbohydrates. c) 20 g of carbohydrates. d) 25 g of carbohydrates.

b) 15 g of carbohydrates. Explanation: The nurse should instruct the client to administer 1 unit of insulin for every 15 g of carbohydrates.

Category: Endocrine and Metabolic Disorders A nurse is teaching a client with diabetes mellitus about self-management of his condition. The nurse should instruct the client to administer 1 unit of insulin for every: a)10 g of carbohydrates. b)15 g of carbohydrates. c)20 g of carbohydrates. d)25 g of carbohydrates.

b) 15 g of carbohydrates. Reason: The nurse should instruct the client to administer 1 unit of insulin for every 15 g of carbohydrates.

Category: Neurosensory Disorders When planning care for a client with a head injury, which position should the nurse include in the care plan to enhance client outcomes? a) Trendelenburg's b) 30-degree head elevation c) Flat d) Side-lying

b) 30-degree head elevation Reason: For clients with increased intracranial pressure (ICP), the head of the bed should be elevated to 30 degrees to promote venous outflow. Trendelenburg's position is contraindicated because it can raise ICP. Flat or neutral positioning is indicated when elevating the head of the bed would increase the risk of neck injury or airway obstruction. A side-lying position isn't specifically a therapeutic treatment for increased ICP.

Category: Basic Physical Care The nurse-manager of a home health facility includes which item in the capital budget? a) Salaries and benefits for her staff b) A $1,200 computer upgrade c) Office supplies d) Client-education materials costing $300

b) A $1,200 computer upgrade Explanation: Capital budgets generally include items valued at more than $500. Salaries and benefits are part of the personnel budget. Office supplies and client education materials are part of the operating budget.

Category: Gastrointestinal Disorders A client with inflammatory bowel disease is receiving total parenteral nutrition (TPN). The basic component of the client's TPN solution is most likely to be: a) An isotonic dextrose solution. b) A hypertonic dextrose solution. c) A hypotonic dextrose solution. d) A colloidal dextrose solution.

b) A hypertonic dextrose solution. Reason: The TPN solution is usually a hypertonic dextrose solution. The greater the concentration of dextrose in solution, the greater the tonicity. Hypertonic dextrose solutions are used to meet the body's calorie demands in a volume of fluid that will not overload the cardiovascular system. An isotonic dextrose solution (e.g., 5% dextrose in water) or a hypotonic dextrose solution will not provide enough calories to meet metabolic needs. Colloids are plasma expanders and blood products and are not used in TPN.

Category: Gastrointestinal Disorders A client with an incomplete small-bowel obstruction is to be treated with a Cantor tube. Which of the following measures would most likely be included in the client's care once the Cantor tube has passed into the duodenum? a) Maintain bed rest with bathroom privileges. b) Advance the tube 2 to 4 inches at specified times. c) Avoid frequent mouth care. d) Provide ice chips for the client to suck.

b) Advance the tube 2 to 4 inches at specified times. Explanation: Once the intestinal tube has passed into the duodenum, it is usually advanced as ordered 2 to 4 inches every 30 to 60 minutes. This, along with gravity and peristalsis, enables passage of the tube forward. The client is encouraged to walk, which also facilitates tube progression. A client with an intestinal tube needs frequent mouth care to stimulate saliva secretion, to maintain a healthy oral cavity, and to promote comfort regardless of where the tube is placed in the intestine. Ice chips are contraindicated because hypotonic fluid will draw extra fluid into an already distended bowel.

Category: Basic Physical Assessment A nurse is assessing a client's pulse. Which pulse feature should the nurse document? a) Timing in the cycle b) Amplitude c) Pitch d) Intensity

b) Amplitude Explanation: The nurse should document the rate, rhythm, and amplitude, such as weak or bounding, of a client's pulse. Pitch, timing, and intensity aren't associated with pulse assessment.

Category: Basic Physical Assessment A nurse is assessing a client's pulse. Which pulse feature should the nurse document? a)Timing in the cycle b)Amplitude c)Pitch d)Intensity

b) Amplitude Reason: The nurse should document the rate, rhythm, and amplitude, such as weak or bounding, of a client's pulse.Pitch, timing, and intensity aren't associated with pulse assessment.

Category: The Neonate A woman who has recently immigrated from Africa who delivered a term neonate a short time ago requests that a "special bracelet" be placed on the baby's wrist. The nurse should: a) Tell the mother that the bracelet is not recommended for cleanliness reasons. b) Apply the bracelet on the neonate's wrist as the mother requests. c) Place the bracelet on the neonate, limiting its use to when the neonate is with the mother. d) Recommend that the mother wait until she is discharged to apply the bracelet.

b) Apply the bracelet on the neonate's wrist as the mother requests. Explanation: The nurse should abide by the mother's request and place the bracelet on the neonate. In some cultures, amulets and other special objects are viewed as good luck symbols. By allowing the bracelet, the nurse demonstrates culturally sensitive care, promoting trust. The neonate can wear the bracelet while with the mother or in the nursery. The bracelet can be used while the neonate is being bathed, or if necessary and acceptable to the client removed and replaced afterward.

Category: Antepartum Period A nurse is assessing the legs of a client who's 36 weeks pregnant. Which finding should the nurse expect? a) Absent pedal pulses b) Bilateral dependent edema c) Sluggish capillary refill d) Unilateral calf enlargement

b) Bilateral dependent edema Reason: As the uterus grows heavier during pregnancy, femoral venous pressure rises, leading to bilateral dependent edema. Factors interfering with venous return, such as sitting or standing for long periods, contribute to edema. Absence of pedal pulses and sluggish capillary refill signal inadequate circulation to the legs — an unexpected finding during pregnancy. Unilateral calf enlargement, also an abnormal finding, may indicate thrombosis.

Category: Antepartum Period A nurse is assessing the legs of a client who's 36 weeks pregnant. Which finding should the nurse expect? a)Absent pedal pulses b)Bilateral dependent edema c)Sluggish capillary refill d)Unilateral calf enlargement

b) Bilateral dependent edema Reason: As the uterus grows heavier during pregnancy, femoral venous pressure rises, leading to bilateral dependent edema. Factors interfering with venous return, such as sitting or standing for long periods, contribute to edema. Absence of pedal pulses and sluggish capillary refill signal inadequate circulation to the legs — an unexpected finding during pregnancy. Unilateral calf enlargement, also an abnormal finding, may indicate thrombosis.

Category: Gastrointestinal Disorders A client with cholecystitis is taking Propantheline bromide (Pro-Banthine). The expected outcome of this drug is: a) Increased bile production. b) Decreased biliary spasm. c) Absence of infection. d) Relief from nausea.

b) Decreased biliary spasm. Explanation: Propantheline bromide is an anticholinergic used to decrease biliary spasm. Decreasing biliary spasm helps to reduce pain in cholecystitis. Propantheline does not increase bile production or have an antiemetic effect, and it is not effective in treating infection.

Category: Basic Physical Assessment The nurse is assessing a client's testes. Which of the following findings indicate the testes are normal? a) Soft. b) Egg-shaped. c) Spongy. d) Lumpy.

b) Egg-shaped. Reason: Normal testes feel smooth, egg-shaped, and firm to the touch, without lumps. The surface should feel smooth and rubbery. The testes should not be soft or spongy to the touch. Testicular malignancies are usually nontender, nonpainful hard lumps. Lumps, swelling, nodules, or signs of inflammation should be reported to the physician.

Category: Basic Physical Assessment The nurse is assessing a client's testes. Which of the following findings indicate the testes are normal? a) Soft b) Egg-shaped c) Spongy d) Lumpy

b) Egg-shaped. Reason: Normal testes feel smooth, egg-shaped, and firm to the touch, without lumps. The surface should feel smooth and rubbery. The testes should not be soft or spongy to the touch. Testicular malignancies are usually nontender, nonpainful hard lumps. Lumps, swelling, nodules, or signs of inflammation should be reported to the physician.

Category: Psychotic Disorders Which condition or characteristic is related to the cluster of symptoms associated with disorganized schizophrenia? a) Odd beliefs b) Flat affect c) Waxy flexibility d) Systematized delusions

b) Flat affect Reason: Flat affect (the lack of facial or behavioral manifestations of emotion) is related to disorganized schizophrenia. Other characteristics of disorganized schizophrenia include incoherence, loose associations, and disorganized behavior. Paranoid residual type schizophrenia is characterized by odd beliefs, unusual perceptions, and systematized delusions. Waxy flexibility, or maintaining the position the client is placed in, is seen in catatonic schizophrenia.

Category: Psychotic Disorders A client with bipolar disorder, manic phase, just sat down to watch television in the lounge. As the nurse approaches the lounge area, the client states, "The sun is shining. Where is my son? I love Lucy. Let's play ball." The client is displaying: a) Concreteness. b) Flight of ideas. c) Depersonalization. d) Use of neologisms.

b) Flight of ideas. Reason: The client is demonstrating flight of ideas, or the rapid, unconnected, and often illogical progression from one topic to another. Concreteness involves interpreting another person's words literally. Depersonalization refers to feelings of strangeness concerning the environment or the self. A neologism is a word made up by a client.

Category: School-age Child After staying several hours with her 9-year-old daughter who is admitted to the hospital with an asthma attack, the mother leaves to attend to her other children. The child exhibits continued signs and symptoms of respiratory distress. Which of the following findings should lead the nurse to believe the child is experiencing anxiety? a) Not able to get comfortable. b) Frequent requests for someone to stay in the room. c) Inability to remember her exact address. d) Verbalization of a feeling of tightness in her chest.

b) Frequent requests for someone to stay in the room.

Category: Postpartum Period The nurse is caring for several mother-baby couplets. In planning the care for each of the couplets, which mother would the nurse expect to have the most severe afterbirth pains? a) G 4, P 1 client who is breastfeeding her infant. b) G 3, P 3 client who is breastfeeding her infant. c) G 2, P 2 cesarean client who is bottle-feeding her infant. d) G 3, P 3 client who is bottle-feeding her infant.

b) G 3, P 3 client who is breastfeeding her infant. Reason: The major reasons for afterbirth pains are breast-feeding, high parity, overdistended uterus during pregnancy, and a uterus filled with blood clots. Physiologically, afterbirth pains are caused by intermittent contraction and relaxation of the uterus. These contractions are stronger in multigravidas in order to maintain a contracted uterus. The release of oxytocin when breast-feeding also stimulates uterine contractions. There is no data to suggest any of these clients has had an overdistended uterus or currently has clots within the uterus. The G 3, P 3 client who is breast-feeding has the highest parity of the clients listed, which—in addition to breast-feeding—places her most at risk for afterbirth pains. The G 2, P 2 postcesarean client may have cramping but it should be less than the G 3, P 3 client. The G 3, P 3 client who is bottle-feeding would be at risk for afterbirth pains because she has delivered several children, but her choice to bottle-feed reduces her risk of pain.

Category: Basic Physical Care As a nurse helps a client ambulate, the client says, "I had trouble sleeping last night." Which action should the nurse take first? a) Recommending warm milk or a warm shower at bedtime b) Gathering more information about the client's sleep problem c) Determining whether the client is worried about something d) Finding out whether the client is taking medication that may impede sleep

b) Gathering more information about the client's sleep problem Reason: The nurse first should determine what the client means by "trouble sleeping." The nurse lacks sufficient information to recommend warm milk or a warm shower or to make inferences about the cause of the sleep problem, such as worries or medication use.

Category: Infant Before placement of a ventriculoperitoneal shunt for hydrocephalus, an infant is irritable, lethargic, and difficult to feed. To maintain the infant's nutritional status, which of the following actions would be most appropriate? a) Feeding the infant just before doing any procedures. b) Giving the infant small, frequent feedings. c) Feeding the infant in a horizontal position. d) Scheduling the feedings for every 6 hours.

b) Giving the infant small, frequent feedings. Explanation: An infant with hydrocephalus is difficult to feed because of poor sucking, lethargy, and vomiting, which are associated with increased intracranial pressure. Small, frequent feedings given at times when the infant is relaxed and calm are tolerated best. Feeding an infant before any procedure is inappropriate because the stress of the procedure may lead to vomiting. Ideally, the infant should be held in a slightly vertical position when feeding to prevent backflow of formula into the eustachian tubes and subsequent development of ear infections. Most infants are fed on demand every 3 to 4 hours.

Category: Medication and I.V. Administration Total parenteral nutrition (TPN) is prescribed for a client who has recently had a significant small and large bowel resection and is currently not taking anything by mouth. The nurse should: a) Administer TPN through a nasogastric or gastrostomy tube. b) Handle TPN using strict aseptic technique. c) Auscultate for bowel sounds prior to administering TPN. d) Designate a peripheral intravenous (IV) site for TPN administration.

b) Handle TPN using strict aseptic technique. Reason: TPN is hypertonic, high-calorie, high-protein, intravenous (IV) fluid that should be provided to clients without functional gastrointestinal tract mobility, to better support their metabolic needs and to support optimal nutrition and healing. TPN is ordered once daily, based on the client's current electrolyte and fluid balance and must be handled with strict aseptic technique (because of its high glucose content, it is a perfect medium for bacterial growth). Also, because of its high tonicity, TPN must be administered via central venous access, not a peripheral IV line.

Category: Mood, Adjustment, and Dementia Disorders A nurse is monitoring a client receiving tranylcypromine sulfate (Parnate). Which serious adverse reaction can occur with high dosages of this monoamine oxidase (MAO) inhibitor? a) Hypotensive episodes b) Hypertensive crisis c) Muscle flaccidity d) Hypoglycemia

b) Hypertensive crisis Reason: The most serious adverse reaction associated with high doses of MAO inhibitors is hypertensive crisis, which can lead to death. Although not a crisis, orthostatic hypotension is also common and may lead to syncope with high doses. Muscle spasticity (not flaccidity) is associated with MAO inhibitor therapy. Hypoglycemia isn't an adverse reaction of MAO inhibitors.

Category: Intrapartum Period A client with Rh isoimmunization gives birth to a neonate with an enlarged heart and severe, generalized edema. The neonate is immediately transferred to the neonatal intensive care unit. Which nursing diagnosis is most appropriate for the client? a) Ineffective denial related to a socially unacceptable infection b) Impaired parenting related to the neonate's transfer to the intensive care unit c) Deficient fluid volume related to severe edema d) Fear related to removal and loss of the neonate by statute

b) Impaired parenting related to the neonate's transfer to the intensive care unit

Category: Basic Physical Care The nurse observes that the right eye of an unconscious client does not close completely. Which nursing intervention is most appropriate? a) Have the client wear eyeglasses at all times. b) Lightly tape the eyelid shut. c) Instill artificial tears once every shift. d) Clean the eyelid with a washcloth every shift.

b) Lightly tape the eyelid shut. Reason: When the blink reflex is absent or the eyes do not close completely, the cornea may become dry and irritated. Corneal abrasion can occur. Taping the eye closed will prevent injury.

Category: Basic Psychosocial Needs An Arab client with pneumonia has been admitted to the health care facility. What should the nurse avoid while conducting the interview of the client? a) Giving a light handshake. b) Maintaining eye contact. c) Asking about the client's symptoms. d) Asking about the client's medical history.

b) Maintaining eye contact. Explanation: While interviewing an Arab client, the nurse should avoid maintaining eye contact. In Arab culture, maintaining eye contact is sexually suggestive; if the nurse does so during the interview, it may give the wrong message to the client. However, the nurse may give a light handshake or ask about the client's personal life and medical history during the interview.

Category: School-age Child A 10-year-old with glomerulonephritis reports a headache and blurred vision. The nurse should immediately: a) Put the client to bed. b) Obtain the child's blood pressure. c) Notify the physician. d) Administer acetaminophen (Tylenol).

b) Obtain the child's blood pressure. Reason: Hypertension occurs with acute glomerulonephritis. The symptoms of headache and blurred vision may indicate an elevated blood pressure. Hypertension in acute glomerulonephritis occurs due to the inability of the kidneys to remove fluid and sodium; the fluid is reabsorbed, causing fluid volume excess. The nurse must verify that these symptoms are due to hypertension. Calling the physician before confirming the cause of the symptoms would not assist the physician in his treatment. Putting the client to bed may help treat an elevated blood pressure, but first the nurse must establish that high blood pressure is the cause of the symptoms. Administering Tylenol for high blood pressure is not recommended.

Category: School-age Child A 10-year-old with glomerulonephritis reports a headache and blurred vision. The nurse should immediately: a)Put the client to bed. b)Obtain the child's blood pressure. c)Notify the physician. d)Administer acetaminophen (Tylenol).

b) Obtain the child's blood pressure. Reason: Hypertension occurs with acute glomerulonephritis. The symptoms of headache and blurred vision may indicate an elevated blood pressure. Hypertension in acute glomerulonephritis occurs due to the inability of the kidneys to remove fluid and sodium; the fluid is reabsorbed, causing fluid volume excess. The nurse must verify that these symptoms are due to hypertension. Calling the physician before confirming the cause of the symptoms would not assist the physician in his treatment. Putting the client to bed may help treat an elevated blood pressure, but first the nurse must establish that high blood pressure is the cause of the symptoms. Administering Tylenol for high blood pressure is not recommended.

Category: School-age Child Which of the following measures should the nurse include in the care plan for a child who is receiving high-dose methotrexate (amethopterin) therapy? a) Keeping the child in a fasting state. b) Obtaining a white blood cell (WBC) count. c) Preparing for radiography of the spinal canal. d) Collecting a specimen for urinalysis.

b) Obtaining a white blood cell (WBC) count. Explanation: Methotrexate is not highly toxic in low doses but may cause severe leukopenia at higher doses. It is customary and recommended for blood tests to be done before therapy to provide a baseline from which to study the effects of the drug on WBC count. Maintaining a fasting state, radiography of the spinal canal, and urinalysis are not necessary when this drug is administered.

Category: School-age Child Which of the following measures should the nurse include in the care plan for a child who is receiving high-dose methotrexate (amethopterin) therapy? a) Keeping the child in a fasting state. b) Obtaining a white blood cell (WBC) count. c) Preparing for radiography of the spinal canal. d) Collecting a specimen for urinalysis.

b) Obtaining a white blood cell (WBC) count. Reason: Methotrexate is not highly toxic in low doses but may cause severe leukopenia at higher doses. It is customary and recommended for blood tests to be done before therapy to provide a baseline from which to study the effects of the drug on WBC count. Maintaining a fasting state, radiography of the spinal canal, and urinalysis are not necessary when this drug is administered.

Category: Intrapartum Period The nurse is caring for a multigravid client who speaks little English. As the nurse enters the client's room, the nurse observes the client squatting on the bed and the fetal head crowning. After calling for assistance and helping the client lie down, which of the following actions should the nurse do next? a) Tell the client to push between contractions. b) Provide gentle support to the fetal head. c) Apply gentle upward traction on the neonate's anterior shoulder. d) Massage the perineum to stretch the perineal tissues.

b) Provide gentle support to the fetal head. Reason: During a precipitous delivery, after calling for assistance and helping the client lie down, the nurse should provide support to the fetal head to prevent it from coming out. It is not appropriate to tell the client to push between contractions because this may lead to lacerations. The shoulder should be delivered by applying downward traction until the anterior shoulder appears fully at the introitus, then upward pressure to lift out the other shoulder. Priority should be given to safe delivery of the infant over protecting the perineum by massage.

Category: Mood, Adjustment, and Dementia Disorders A client who took an overdose of Tylenol in a suicide attempt is transferred overnight to the psychiatric inpatient unit from the intensive care unit. The night shift nurse called the primary health care provider on call to obtain initial prescriptions. The primary health care provider prescribes the typical routine medications for clients on this unit: Milk of Magnesia, Maalox and Tylenol as needed. Prior to implementing the prescriptions, the nurse should? a) Ask the primary health care provider about holding all the client's PM prescriptions. b) Question the primary health care provider about the Tylenol prescription. c) Request a prescription for a medication to relieve agitation. d) Suggest the primary health care provider write a prescription for intravenous fluids.

b) Question the primary health care provider about the Tylenol prescription. Reason: The nurse should question the Tylenol order because the client overdosed on Tylenol, and that analgesic would be contraindicated as putting further stress on the liver. There is no need to hold the PM Milk of Magnesia or Maalox. There is no indication that the client is agitated or needs medication for agitation. There is little likelihood that the client needs an IV after being transferred out of an intensive care unit, as the client will be able to take oral fluids.

Category: Respiratory Disorders The nurse has administered aminophylline to a client with emphysema. The medication is effective when there is: a) Relief from spasms of the diaphragm. b) Relaxation of smooth muscles in the bronchioles. c) Efficient pulmonary circulation. d) Stimulation of the medullary respiratory center.

b) Relaxation of smooth muscles in the bronchioles. Reason: Aminophylline, a bronchodilator that relaxes smooth muscles in the bronchioles, is used in the treatment of emphysema to improve ventilation by dilating the bronchioles. Aminophylline does not have an effect on the diaphragm or the medullary respiratory center and does not promote pulmonary circulation.

Category: Postpartum Period A primiparous client at 4 hours after a vaginal delivery and manual removal of the placenta voids for the first time. The nurse palpates the fundus, noting it to be 1 cm above the umbilicus, slightly firm, and deviated to the left side, and notes a moderate amount of lochia rubra. The nurse notifies the physician based on the interpretation that the assessment indicates which of the following? a) Perineal lacerations. b) Retained placental fragments. c) Cervical lacerations. d) Urine retention.

b) Retained placental fragments. Explanation: At 4 hours postpartum, the fundus should be midline and at the level of the umbilicus. Whenever the placenta is manually removed after delivery, there is a possibility that all of the placenta has not been removed. Sometimes small pieces of the placenta are retained, a common cause of late postpartum hemorrhage. The client is exhibiting signs and symptoms associated with retained placental fragments. The client will continue to bleed until the fragments are expelled. Perineal and cervical lacerations are characterized by bright red bleeding and a firmly contracted fundus at the level that is expected. Urine retention is characterized by a full bladder, which can be observed by a bulge or fullness just above the symphysis pubis. Also, the client's fundus would be deviated to one side and boggy to the touch.

Category: Postpartum Period A primiparous client at 4 hours after a vaginal delivery and manual removal of the placenta voids for the first time. The nurse palpates the fundus, noting it to be 1 cm above the umbilicus, slightly firm, and deviated to the left side, and notes a moderate amount of lochia rubra. The nurse notifies the physician based on the interpretation that the assessment indicates which of the following? a) Perineal lacerations. b) Retained placental fragments. c) Cervical lacerations. d) Urine retention.

b) Retained placental fragments. Reason: At 4 hours postpartum, the fundus should be midline and at the level of the umbilicus. Whenever the placenta is manually removed after delivery, there is a possibility that all of the placenta has not been removed. Sometimes small pieces of the placenta are retained, a common cause of late postpartum hemorrhage. The client is exhibiting signs and symptoms associated with retained placental fragments. The client will continue to bleed until the fragments are expelled. Perineal and cervical lacerations are characterized by bright red bleeding and a firmly contracted fundus at the level that is expected. Urine retention is characterized by a full bladder, which can be observed by a bulge or fullness just above the symphysis pubis. Also, the client's fundus would be deviated to one side and boggy to the touch.

Category: Substance Abuse, Eating Disorders, Impulse Control Disorders After a dose-response test, the client with an overdose of barbiturates receives pentobarbital sodium (Nembutal) at a nonintoxicating maintenance level for 2 days and at decreasing dosages thereafter. This regimen is effective in the client does not develop: a) Psychosis. b) Seizures. c) Hypotension. d) Hypothermia.

b) Seizures. Explanation: Generalized seizures may occur on the second or third day of withdrawal from barbiturates. Without treatment, the seizures may be fatal. Psychosis is a possibility but is not fatal and will not be prevented by the pentobarbital sodium regimen. Orthostatic hypotension is possible but is unlikely to be fatal; it is also not treatable by the pentobarbital sodium regimen. Hyperthermia, rather than hypothermia, occurs during withdrawal.

Category: Substance Abuse, Eating Disorders, Impulse Control Disorders After a dose-response test, the client with an overdose of barbiturates receives pentobarbital sodium (Nembutal) at a nonintoxicating maintenance level for 2 days and at decreasing dosages thereafter. This regimen is effective in the client does not develop: a) Psychosis. b) Seizures. c) Hypotension. d) Hypothermia.

b) Seizures. Reason: Generalized seizures may occur on the second or third day of withdrawal from barbiturates. Without treatment, the seizures may be fatal. Psychosis is a possibility but is not fatal and will not be prevented by the pentobarbital sodium regimen. Orthostatic hypotension is possible but is unlikely to be fatal; it is also not treatable by the pentobarbital sodium regimen. Hyperthermia, rather than hypothermia, occurs during withdrawal.

Category: Medication and I.V. Administration A woman is taking oral contraceptives. The nurse teaches the client to report which of the following danger signs? a) Breakthrough bleeding. b) Severe calf pain. c) Mild headache. d) Weight gain of 3 lb.

b) Severe calf pain. Reason: Women who take oral contraceptives are at increased risk for thromboembolic conditions. Severe calf pain needs to be investigated as a potential sign of deep vein thrombosis. Breakthrough bleeding, mild headache, or weight gain may be common benign side effects that accompany oral contraceptive use. Clients may be monitored for these side effects without a change in treatment.

Category: Antepartum Period A multigravid client in labor at 38 weeks' gestation has been diagnosed with Rh sensitization and probable fetal hydrops and anemia. When the nurse observes the fetal heart rate pattern on the monitor, which of the following patterns is most likely? a) Early deceleration pattern. b) Sinusoidal pattern. c) Variable deceleration pattern. d) Late deceleration pattern.

b) Sinusoidal pattern. Reason: The fetal heart rate of a multipara diagnosed with Rh sensitization and probable fetal hydrops and anemia will most likely demonstrate a sinusoidal pattern that resembles a sine wave. It has been hypothesized that this pattern reflects an absence of autonomic nervous control over the fetal heart rate resulting from severe hypoxia. This client will most likely require a cesarean delivery to improve the fetal outcome. Early decelerations are associated with head compression; variable decelerations are associated with cord compression; and late decelerations are associated with poor placental perfusion.

Category: Infant The nurse is assessing the development of a 7-month-old. The child should be able to: a) Play pat-a-cake. b) Sit without support. c) Say two words. d) Wave bye-bye.

b) Sit without support. Reason: Sit without support. The majority of infancts (90%) can sit without support by 7 months of age.

Category: Preschooler A dehydrated 3 year old has vomited three times in the last hour and continues to have frequent diarrhea. The child was admitted 2 days ago with gastroenteritis caused by rotavirus. The child weighs 22 kg, has a normal saline lock in the right hand, and has had 30 ml of urine output in the last 4 hours. Using the SBAR (Situation-Background-Assessment-Recommendation) technique for communication, the nurse calls the primary healthcare provider with a recommendation for: a) Giving a dose of loperaminde (Immodium). b) Starting a fluid bolus of normal saline. c) Beginning an intravenous (IV) antibiotic. d) Establishing a Foley catheter.

b) Starting a fluid bolus of normal saline. Explanation: The child is dehydrated, cannot retain oral fluids, and continues to have diarrhea. A normal saline bolus should be given followed by maintenance IV fluids. Anti-diarrheal medications are not recommended for children and will prolong the illness. The child has gastroenteritis caused by a viral illness. IV antibiotics are not indicated for viral illnesses.

Category: Preschooler A dehydrated 3 year old has vomited three times in the last hour and continues to have frequent diarrhea. The child was admitted 2 days ago with gastroenteritis caused by rotavirus. The child weighs 22 kg, has a normal saline lock in the right hand, and has had 30 ml of urine output in the last 4 hours. Using the SBAR (Situation-Background-Assessment-Recommendation) technique for communication, the nurse calls the primary healthcare provider with a recommendation for: a) Giving a dose of loperaminde (Immodium). b) Starting a fluid bolus of normal saline. c) Beginning an intravenous (IV) antibiotic. d) Establishing a Foley catheter.

b) Starting a fluid bolus of normal saline. Reason: The child is dehydrated, cannot retain oral fluids, and continues to have diarrhea. A normal saline bolus should be given followed by maintenance IV fluids. Anti-diarrheal medications are not recommended for children and will prolong the illness. The child has gastroenteritis caused by a viral illness. IV antibiotics are not indicated for viral illnesses.

Category: Oncologic Disorders A 45-year-old single mother of three teenaged boys has metastatic breast cancer. Her parents live 750 miles away and have only been able to visit twice since her initial diagnosis 14 months ago. The progression of her disease has forced the client to consider high-dose chemotherapy. She is concerned about her children's welfare during the treatment. When assessing the client's present support systems, the nurse will be most concerned about the potential problems with: a) Denial as a primary coping mechanism. b) Support systems and coping strategies. c) Decision-making abilities. d) Transportation and money for the boys.

b) Support systems and coping strategies. Explanation: The client's resources for coping with the emotional and practical needs of herself and her family need to be assessed because usual coping strategies and support systems are often inadequate in especially stressful situations. The nurse may be concerned with the client's use of denial, decision-making abilities, and ability to pay for transportation; however, the client's support systems will be of more importance in this situation.

Category: Oncologic Disorders A 45-year-old single mother of three teenaged boys has metastatic breast cancer. Her parents live 750 miles away and have only been able to visit twice since her initial diagnosis 14 months ago. The progression of her disease has forced the client to consider high-dose chemotherapy. She is concerned about her children's welfare during the treatment. When assessing the client's present support systems, the nurse will be most concerned about the potential problems with: a) Denial as a primary coping mechanism. b) Support systems and coping strategies. c) Decision-making abilities. d) Transportation and money for the boys.

b) Support systems and coping strategies. Reason: The client's resources for coping with the emotional and practical needs of herself and her family need to be assessed because usual coping strategies and support systems are often inadequate in especially stressful situations. The nurse may be concerned with the client's use of denial, decision-making abilities, and ability to pay for transportation; however, the client's support systems will be of more importance in this situation.

Category: Basic Physical Care Which scenario complies with Health Insurance Portability and Accountability Act of 1996 (HIPAA) regulations? a) Two nurses in the cafeteria are discussing a client's condition. b) The health care team is discussing a client's care during a formal care conference. c) A nurse checks the computer for the laboratory results of a neighbor who has been admitted to another floor. d) A nurse talks with her spouse about a client's condition.

b) The health care team is discussing a client's care during a formal care conference. Reason: To provide interdisciplinary continuity of care, nurses must share relevant information during client care conferences. Nurses discussing information in the cafeteria may be overheard; this indiscretion violates HIPAA regulations. Looking up laboratory results for a neighbor is a HIPAA violation, as is discussing a client's condition with one's spouse.

Category: Genitourinary Disorders After surgery for an ileal conduit, the nurse should closely assess the client for the occurrence of which of the following complications related to pelvic surgery? a) Peritonitis. b) Thrombophlebitis. c) Ascites. d) Inguinal hernia.

b) Thrombophlebitis. Reason: After pelvic surgery, there is an increased chance of thrombophlebitis owing to the pelvic manipulation that can interfere with circulation and promote venous stasis. Peritonitis is a potential complication of any abdominal surgery, not just pelvic surgery. Ascites is most frequently an indication of liver disease. Inguinal hernia may be caused by an increase in intra-abdominal pressure or a congenital weakness of the abdominal wall; ventral hernia occurs at the site of a previous abdominal incision.

Category: Respiratory Disorders A nurse assesses a client's respiratory status. Which observation indicates that the client is having difficulty breathing? a) Diaphragmatic breathing b) Use of accessory muscles c) Pursed-lip breathing d) Controlled breathing

b) Use of accessory muscles Reason: The use of accessory muscles for respiration indicates the client is having difficulty breathing. Diaphragmatic and pursed-lip breathing are two controlled breathing techniques that help the client conserve energy.

Category: Infant A 10-month-old child has cold symptoms. The mother asks how she can clear the infant's nose. Which of the following would be the nurse's best recommendation? a) Use a cool air vaporizer with plain water. b) Use saline nose drops and then a bulb syringe. c) Blow into the child's mouth to clear the infant's nose. d) Administer a nonprescription vasoconstrictive nose spray.

b) Use saline nose drops and then a bulb syringe. Reason: although a cool air vaporizer may be recommended to humidify the environment, using saline nose drops and then a bulb syringe before meals and at nap and bed times will allow the child to breathe more easily. Saline helps to loosen secretions and keep the mucous membranes moist. The bulb syringe then gently aids in removing the loosened secretions. Blowing into the child's mouth to clear the nose introduces more organisms to the child. A nonprescription vasoconstrictive nasal spray is not recommended for infants because if the spray in used for longer than 3 days a rebound effect with increased inflammation occurs.

Category: Cardiovascular Disorders When assessing a client for early septic shock, the nurse should assess the client for which of the following? a) Cool, clammy skin. b) Warm, flushed skin. c) Increased blood pressure. d) Hemorrhage.

b) Warm, flushed skin. Reason: Warm, flushed skin from a high cardiac output with vasodilation occurs in warm shockor the hyperdynamic phase (first phase) of septic shock. Other signs and symptoms of early septic shock include fever with restlessness and confu- sion; decreased blood pressure with tachypnea and tachycardia; increased or normal urine output; and nausea and vomiting or diarrhea. Cool, clammy skin occurs in the hypodynamic or cold phase (later phase). Hemorrhage is not a factor in septic shock.

Category: Musculoskeletal Disorders After surgery to treat a hip fracture, a client returns from the postanesthesia care unit to the medical-surgical unit. Postoperatively, how should the nurse position the client? a) With the affected hip flexed acutely b) With the leg on the affected side abducted c) With the leg on the affected side adducted d) With the affected hip rotated externally

b) With the leg on the affected side abducted Explanation: The nurse must keep the leg on the affected side abducted at all times after hip surgery to prevent accidental dislodgment of the affected hip joint. Placing a pillow or an A-frame between the legs helps maintain abduction and reminds the client not to cross the legs. The nurse should avoid acutely flexing the client's affected hip (for example, by elevating the head of the bed excessively), adducting the leg on the affected side (such as by moving it toward the midline), or externally rotating the affected hip (such as by removing support along the outer side of the leg) because these positions may cause dislocation of the injured hip joint.

Category: Musculoskeletal Disorders After surgery to treat a hip fracture, a client returns from the postanesthesia care unit to the medical-surgical unit. Postoperatively, how should the nurse position the client? a) With the affected hip flexed acutely b) With the leg on the affected side abducted c) With the leg on the affected side adducted d) With the affected hip rotated externally

b) With the leg on the affected side abducted Reason: The nurse must keep the leg on the affected side abducted at all times after hip surgery to prevent accidental dislodgment of the affected hip joint. Placing a pillow or an A-frame between the legs helps maintain abduction and reminds the client not to cross the legs. The nurse should avoid acutely flexing the client's affected hip (for example, by elevating the head of the bed excessively), adducting the leg on the affected side (such as by moving it toward the midline), or externally rotating the affected hip (such as by removing support along the outer side of the leg) because these positions may cause dislocation of the injured hip joint.

Category: The Nursing Process A parent brings a 5-year-old child to a vaccination clinic to prepare for school entry. The nurse notes that the child has not had any vaccinations since 4 months of age. To determine the current evidence for best practices for scheduling missed vaccinations the nurse should: a) Ask the primary care provider. b) Check the website at the Center for Disease Control and Prevention (CDC). c) Read the vaccine manufacturer's insert. d) Contact the pharmacist.

b) check the website at the Center for Disease Control and Prevention (CDC) Reason: The CDC is the federal body that is ultimately responsible for vaccination recommendations for adults and children. A division of the CDC, the Advisory Committee on Immunization Practices, reviews vaccination evidence and updates recommendation on a yearly basis. The CDC publishes current vaccination catch-up schedules that are readily available on their website. The lack of vaccinations is a strong indicator that the child probably does not have a primary care provider. If consulted, the pharmacist would most likely have to review the CDC guidelines that are equally available to the nurse. Reading the manufacturer's inserts for multiple vaccines would be time consuming and synthesis of the information could possibly lead to errors.

Category: The Neonate A primigravid client gives birth to a full-term girl. When teaching the client and her partner how to change their neonate's diaper, the nurse should instruct them to: a) fold a cloth diaper so that a double thickness covers the front. b) clean and dry the neonate's perineal area from front to back. c) place a disposable diaper over a cloth diaper to provide extra protection. d) position the neonate so that urine will fall to the back of the diaper.

b) clean and dry the neonate's perineal area from front to back. Explanation: When changing a female neonate's diaper, the caregiver should clean the perineal area from front to back to prevent infection and then dry the area thoroughly to minimize skin breakdown. For a male, the caregiver should clean and dry under and around the scrotum. Because of anatomic factors, a female's diaper should have the double thickness toward the back. The diaper, not the neonate, should be positioned properly. Placing a disposable diaper over a cloth diaper isn't necessary. The direction of urine flow can't be ensured.

Category: Gastrointestinal Disorders A client with gastroenteritis is admitted to an acute care facility and presents with severe dehydration and electrolyte imbalances. Diagnostic tests reveal the Norwalk virus as the cause of gastroenteritis. Based on this information, the nurse knows that: a) the client requires an antiviral agent. b) enteric precautions must be continued. c) enteric precautions can be discontinued. d) the client's infection may be caused by droplet transmission.

b) enteric precautions must be continued. RATIONALES: The nurse must continue enteric precautions for a client with gastroenteritis caused by the Norwalk virus because this virus is transmitted by the fecal-oral route. No safe and effective antiviral agent is available specifically for treating viral gastroenteritis. The Norwalk virus isn't transmitted by droplets.

Category: Mood, Adjustment, and Dementia Disorders The major goal of therapy in crisis intervention is to: a) withdraw from the stress. b) resolve the immediate problem. c) decrease anxiety. d) provide documentation of events.

b) resolve the immediate problem. Reason: During a period of crisis, the major goal is to resolve the immediate problem, with hopes of getting the individual to the level of functioning that existed before the crisis or to a higher level of functioning. Withdrawing from stress doesn't address the immediate problem and isn't therapeutic. The client's anxiety will decrease after the immediate problem is resolved. Providing support and safety are necessary interventions while working toward accomplishing the goal. Documentation is necessary for maintaining accurate records of treatment; it isn't a major goal.

Category: Preschooler After discussing asthma as a chronic condition, which of the following statements by the father of a child with asthma best reflects the family's positive adjustment to this aspect of the child's disease? a) "We try to keep him happy at all costs; otherwise, he has an asthma attack." b) "We keep our child away from other children to help cut down on infections." c) "Although our child's disease is serious, we try not to let it be the focus of our family." d) "I'm afraid that when my child gets older, he won't be able to care for himself like I do."

c) "Although our child's disease is serious, we try not to let it be the focus of our family." Reason: Positive adjustment to a chronic condition requires placing the child's illness in its proper perspective. Children with asthma need to be treated as normally as possible within the scope of the limitations imposed by the illness. They also need to learn how to manage exacerbations and then resume as normal a life as possible . Trying to keep the child happy at all costs is inappropriate and can lead to the child's never learning how to accept responsibility for behavior and get along with others. Although minimizing the child's risk for exposure to infections is important , the child needs to be with his or her peers to ensure appropriate growth and developmentChildren with a chronic illness need to be involved in their care so that they can learn to manage it. Some parents tend to overprotect their child with a chronic illness. This overprotectiveness may cause a child to have an exaggerated feeling of importance or later, as an adolescent, to rebel against the overprotectiveness and the parents.

Category: Preschooler After discussing asthma as a chronic condition, which of the following statements by the father of a child with asthma best reflects the family's positive adjustment to this aspect of the child's disease? a)"We try to keep him happy at all costs; otherwise, he has an asthma attack." b)"We keep our child away from other children to help cut down on infections." c)"Although our child's disease is serious, we try not to let it be the focus of our family." d)"I'm afraid that when my child gets older, he won't be able to care for himself like I do."

c) "Although our child's disease is serious, we try not to let it be the focus of our family." Reason: Positive adjustment to a chronic condition requires placing the child's illness in its proper perspective. Children with asthma need to be treated as normally as possible within the scope of the limitations imposed by the illness. They also need to learn how to manage exacerbations and then resume as normal a life as possible. Trying to keep the child happy at all costs is inappropriate and can lead to the child's never learning how to accept responsibility for behavior and get along with others. Although minimizing the child's risk for exposure to infections is important, the child needs to be with his or her peers to ensure appropriate growth and development. Children with a chronic illness need to be involved in their care so that they can learn to manage it. Some parents tend to overprotect their child with a chronic illness. This overprotectiveness may cause a child to have an exaggerated feeling of importance or later, as an adolescent, to rebel against the overprotectiveness and the parents.

Category: Adolescent A nurse is about to conduct a sexual history for a 16-year-old female who is accompanied by her mother. What is an appropriate question for the nurse to ask this client or her mother? a) "What do you think about having your mother leave the room now?" b) "Mother, do you think your daughter is sexually active?" c) "Mother, I am going to ask you to wait a few minutes in the waiting room now so I can complete the health history with your daughter." d) "The two of you seem like you share everything. I am going to ask questions about sexual history now."

c) "Mother, I am going to ask you to wait a few minutes in the waiting room now so I can complete the health history with your daughter."

Category: Foundations of Psychiatric Nursing The family of an older adult wants their mother to have counseling for depression. During the initial nursing assessment, the client denies the need for counseling. Which of the following comments by the client supports the fact that the client may not need counseling? a) "My doctor just put me on an antidepressant, and I'll be fine in a week or so." b) "My daughter sent me here. She's mad because I don't have the energy to take care of my grandkids." c) "Since I've gotten over the death of my husband, I've had more energy and been more active than before he died." d) "My son got worried because I made this silly comment about wanting to be with my husband in heaven."

c) "Since I've gotten over the death of my husband, I've had more energy and been more active than before he died." Reason: Resolving grief and having increased energy and activity convey good mental health, indicating that counseling is not necessary at this time. Taking an antidepressant or having less energy and involvement with grandchildren reflects possible depression and the need for counseling. Wanting to be with her dead husband suggests possible suicidal ideation that warrants serious further assessment and counseling.

Category: Substance Abuse, Eating Disorders, Impulse Control Disorders A client with alcohol dependency is prescribed a B-complex vitamin. The client states, "Why do I need a vitamin? My appetite is just fine." Which of the following responses by the nurse is most appropriate? a) "Your doctor wants you to take it for at least 4 months." b) "You've been drinking alcohol and eating very little." c) "The vitamin is a nutritional supplement important to your health." d) "The amount of vitamins in the alcohol you drink is very low."

c) "The vitamin is a nutritional supplement important to your health." Reason: Stating that the vitamin is a nutritional supplement important to the client's health is the best response. The client is nutritionally depleted, and the B-complex vitamins produce a calming effect on the irritated central nervous system and prevent anemia, peripheral neuropathy, and Wernicke's encephalopathy. Although the statements about drinking alcohol and eating very little and that there is a low amount of vitamins in the alcohol consumed may be true, they fail to address the client's concerns directly and fail to provide the necessary information, as does telling the client that the doctor wants the client to take the vitamin for 4 months.

Category: Basic Psychosocial Needs A worried mother confides in the nurse that she wants to change physicians because her infant is not getting better. The best response by the nurse is which of the following? a)"This doctor has been on our staff for 20 years." b)"I know you are worried, but the doctor has an excellent reputation." c)"You always have an option to change. Tell me about your concerns." d)"I take my own children to this doctor."

c) "You always have an option to change. Tell me about your concerns." Reason: Asking the mother to talk about her concerns acknowledges the mother's rights and encourages open discussion. The other responses negate the parent's concerns.

Category: Basic Psychosocial Needs A worried mother confides in the nurse that she wants to change physicians because her infant is not getting better. The best response by the nurse is which of the following? a) "This doctor has been on our staff for 20 years." b) "I know you are worried, but the doctor has an excellent reputation." c) "You always have an option to change. Tell me about your concerns." d) "I take my own children to this doctor."

c) "You always have an option to change. Tell me about your concerns." Reason: Asking the mother to talk about her concerns acknowledges the mother's rights and encourages open discussion. The other responses negate the parent's concerns.

Category: Mood, Adjustment, and Dementia Disorders The nurse meets with the client and his wife to discuss depression and the client's medication. Which of the following comments by the wife would indicate that the nurse's teaching about disease process and medications has been effective? a) "His depression is almost cured." b) "He's intelligent and won't need to depend on a pill much longer." c) "It's important for him to take his medication so that the depression will not return or get worse." d) "It's important to watch for physical dependency on Zoloft."

c) "it's important for him to his medication so that the depression will not return or get worse." Reason: Improved balance of neurotransmitters is achieved with medication. Clients with endogenous depression must take antidepressants to prevent a return or worsening of depressive symptoms. Depression is a chronic disease characterized by periods of remission; however, it is not cured. Depression is not dependent on the client's intelligence to will the illness away. Zoloft is not physically addictive.

Category: Medication and I.V. Administration A health care provider orders 0.5 mg of protamine sulfate for a client who is showing signs of bleeding after receiving a 100-unit dose of heparin. The nurse should expect the effects of the protamine sulfate to be noted in which of the following time frames? a) 5 minutes. b) 10 minutes. c) 20 minutes. d) 30 minutes.

c) 20 minutes Reason: A dose of 0.5 mg of protamine sulfate reverses a 100-unit dose of heparin within 20 minutes. The nurse should administer protamine sulfate by I.V. push slowly to avoid adverse effects, such as hypotension, dyspnea, bradycardia, and anaphylaxis

Category: The Nursing Process During the health history interview, which of the following strategies is the most effective for the nurse to use to help clients take an active role in their health care? a) Ask clients to complete a questionnaire. b) Provide clients with written instructions. c) Ask clients for their description of events and for their views concerning past medical care. d) Ask clients if they have any questions.

c) Ask clients for their description of events and for their views concerning past medical care. Reason: One of the best strategies to help clients feel in control is to ask them their view of situations and to respond to what they say. This technique acknowledges that clients' opinions have value and relevance to the interview. It also promotes an active role for clients in the process. Use of a questionnaire or written instructions is a means of obtaining information but promotes a passive client role. Asking whether clients have questions encourages participation but alone it does not acknowledge their views.

Category: Neurosensory Disorders A nurse, a licensed practical nurse (LPN), and a nursing assistant are caring for a group of clients. The nurse asks the nursing assistant to check the pulse oximetry level of a client who underwent laminectomy. The nursing assistant reports that the pulse oximetry reading is 89%. The client Kardex contains an order for oxygen application at 2 L/min should the pulse oximetry level fall below 92%. The nurse is currently assessing a postoperative client who just returned from the postanesthesia care unit. How should the nurse proceed? a) Immediately go the client's room and assess vital signs, administer oxygen at 2 L/minute, and notify the physician. b) Ask the nursing assistant to notify the physician of the low pulse oximetry level. c) Ask the LPN to obtain vital signs and administer oxygen at 2 L/min to the client who underwent laminectomy. d) Complete the assessment of the new client before attending to the client who underwent laminectomy.

c) Ask the LPN to obtain vital signs and administer oxygen at 2 L/min to the client who underwent laminectomy. Explanation: Because it's important to get more information about the client with a decreased pulse oximetry level, the nurse should ask the LPN to obtain vital signs and administer oxygen as ordered. The nurse must attend to the newly admitted client without delaying treatment to the client who is already in her care. The nurse can effectively do this by delegating tasks to an appropriate health team member such as an LPN. The nurse doesn't need to immediately attend to the client with a decreased pulse oximetry level; she may wait until she completes the assessment of the newly admitted client. The physician doesn't need to be notified at this time because an order for oxygen administration is already on record.

Category: Antepartum Period The primary health care provider orders intravenous magnesium sulfate for a primigravid client at 38 weeks' gestation diagnosed with severe preeclampsia. Which of the following medications should the nurse have readily available at the client's bedside? a) Diazepam (Valium). b) Hydralazine (Apresoline). c) Calcium gluconate. d) Phenytoin (Dilantin).

c) Calcium gluconate. Explanation: The client receiving magnesium sulfate intravenously is at risk for possible toxicity. The antidote for magnesium sulfate toxicity is calcium gluconate, which should be readily available at the client's bedside. Diazepam (Valium), used to treat anxiety, usually is not given to pregnant women. Hydralazine (Apresoline) would be used to treat hypertension, and phenytoin (Dilantin) would be used to treat seizures.

Category: Antepartum Period The primary health care provider orders intravenous magnesium sulfate for a primigravid client at 38 weeks' gestation diagnosed with severe preeclampsia. Which of the following medications should the nurse have readily available at the client's bedside? a) Diazepam (Valium). b) Hydralazine (Apresoline). c) Calcium gluconate. d) Phenytoin (Dilantin).

c) Calcium gluconate. Reason: The client receiving magnesium sulfate intravenously is at risk for possible toxicity. The antidote for magnesium sulfate toxicity is calcium gluconate, which should be readily available at the client's bedside. Diazepam (Valium), used to treat anxiety, usually is not given to pregnant women. Hydralazine (Apresoline) would be used to treat hypertension, and phenytoin (Dilantin) would be used to treat seizures.

Category: Cardiovascular Disorders A nurse should monitor a client receiving lidocaine (Xylocaine) for toxicity. Which signs or symptoms in a client suggest lidocaine toxicity? a) Nausea and vomiting b) Pupillary changes c) Confusion and restlessness d) Hypertension

c) Confusion and restlessness Reason: Confusion and restlessness are signs of lidocaine toxicity. Nausea and vomiting may occur with oral administration of mexiletine (Mexitil) or tocainide (Tonocard) — other class IB drugs. Pupillary changes and hypertension aren't signs of lidocaine toxicity, although visual changes and hypotension may occur as adverse reactions to class IB drugs.

Category: Immune and Hematologic Disorders A client receiving a blood transfusion begins to have chills and headache within the first 15 minutes of the transfusion. The nurse should first: a) Administer acetaminophen. b) Take the client's blood pressure. c) Discontinue the transfusion. d) Check the infusion rate of the blood.

c) Discontinue the transfusion.

Category: Immune and Hematologic Disorders A client receiving a blood transfusion begins to have chills and headache within the first 15 minutes of the transfusion. The nurse should first: a) Administer acetaminophen. b) Take the client's blood pressure. c) Discontinue the transfusion. d) Check the infusion rate of the blood.

c) Discontinue the transfusion. Reason: Chills and headache are signs of a febrile, nonhemolytic blood transfusion reaction and the nurse's first action should be to discontinue the transfusion as soon as possible and then notify the physician. Antipyretics and antihistamines may be ordered. The nurse would not administer acetaminophen without an order from the physician. The client's blood pressure should be taken after the transfusion is stopped. Checking the infusion rate of the blood is not a pertinent action; the infusion needs to be stopped regardless of the rate.

Category: Genitourinary Disorders Of the following findings in the client's history, which would be the least likely to have predisposed the client to renal calculi? a) Having had several urinary tract infections in the past 2 years. b) Having taken large doses of vitamin C over the past several years. c) Drinking less than the recommended amount of milk. d) Having been on prolonged bed rest after an accident the previous year.

c) Drinking less than the recommended amount of milk. Reason: A high, rather than low, milk intake predisposes to renal calculi formation, owing to the calcium in milk. Recurrent urinary tract infections are implicated in stone formation as certain bacteria promote stone formation. High daily doses of vitamins C are a risk factor because they can increase the citric acid level. Prolonged immobility is a risk factor for renal calculi because it causes calcium to be released into the bloodstream.

Category: Gastrointestinal Disorders Which of the following laboratory findings are expected when a client has diverticulitis? a) Elevated red blood cell count. b) Decreased platelet count. c) Elevated white blood cell count. d) Elevated serum blood urea nitrogen concentration.

c) Elevated white blood cell count. Reason: Because of the inflammatory nature of diverticulitis, the nurse would anticipate an elevated white blood cell count. The remaining laboratory findings are not associated with diverticulitis. Elevated red blood cell counts occur in clients with polycythemia vera or fluid volume deficit. Decreased platelet counts can occur as a result of aplastic anemias or malignant blood disorders, as an adverse effect of some drugs, and as a result of some heritable conditions. Elevated serum blood urea nitrogen concentration is usually associated with renal conditions.

Category: Postpartum Period Twelve hours after a vaginal delivery with epidural anesthesia, the nurse palpates the fundus of a primiparous client and finds it to be firm, above the umbilicus, and deviated to the right. Which of the following would the nurse do next? a) Document this as a normal finding in the client's record. b) Contact the physician for an order for methylergonovine (Methergine). c) Encourage the client to ambulate to the bathroom and void. d) Gently massage the fundus to expel the clots.

c) Encourage the client to ambulate to the bathroom and void. Reason: At 12 hours postpartum, the fundus normally should be in the midline and at the level of the umbilicus. When the fundus is firm yet above the umbilicus, and deviated to the right rather than in the midline, the client's bladder is most likely distended. The client should be encouraged to ambulate to the bathroom and attempt to void, because a full bladder can prevent normal involution. A firm but deviated fundus above the level of the umbilicus is not a normal finding and if voiding does not return it to midline, it should be reported to the physician. Methylergonovine (Methergine) is used to treat uterine atony. This client's fundus is firm, not boggy or soft, which would suggest atony. Gentle massage is not necessary because there is no evidence of atony or clots.

Category: Genitourinary Disorders A client with chronic renal failure (CRF) has developed faulty red blood cell (RBC) production. The nurse should monitor this client for: a) nausea and vomiting. b) dyspnea and cyanosis. c) fatigue and weakness. d) thrush and circumoral pallor.

c) Fatigue and weakness. Reason: RBCs carry oxygen throughout the body. Decreased RBC production diminishes cellular oxygen, leading to fatigue and weakness. Nausea and vomiting may occur in CRF but don't result from faulty RBC production. Dyspnea and cyanosis are associated with fluid excess, not CRF. Thrush, which signals fungal infection, and circumoral pallor, which reflects decreased oxygenation, aren't signs of CRF.

Category: Basic Psychosocial Needs The health care provider at a prenatal clinic has ordered multivitamins for a woman who is 3 months' pregnant. The client calls the nurse to report that she has gone to the pharmacy to fill her prescription but is unable to buy it as it costs too much. The nurse should refer the client to: a) The charge nurse. b) The hospital finance office. c) Her hospital social worker. d) Her insurance company.

c) Her hospital social worker. Reason: The social worker is available to assist the client in finding services within the community to meet client needs. This individual is able to provide the names of pharmacies within the community that offer generic substitutes or others that utilize the client's insurance plan. The charge nurse of the unit would be able to refer the client to the social worker. The hospital finance office does not handle this type of situation and would refer the client back to the unit. The client's insurance company deals with payments for health care and would refer the client back to the local setting.

Category: Basic Physical Care A physician has ordered penicillin G potassium (Pfizerpen), I.V., for a client with a severe streptococcal infection. A nurse determines that the client may be allergic to penicillin. When considering best practice, what should the nurse's priority intervention be? a) Holding the penicillin G potassium and charting that it was held because the client is allergic b) Administering the penicillin G potassium and staying alert for any reaction c) Holding the penicillin G potassium and notifying the physician that the client may have an allergy to penicillin d) Administering the penicillin G potassium but notifying the pharmacist that the client might experience an allergic reaction

c) Holding the penicillin G potassium and notifying the physician that the client may have an allergy to penicillin Explanation: The nurse should hold the penicillin G potassium, even if the client isn't sure he's allergic to penicillin, and notify the physician so he may order a different antibiotic. Many clients can't act as their own advocates; they rely on nurses to protect their rights. An allergy to penicillin G potassium is suspected, but not comfirmed. Administering penicillin G potassium could cause a life-threatening reaction. Administering the medication, then watching for a reaction or notifying the pharmacist that a reaction might occur, isn't best practice. If a client is allergic to penicillin, a nurse should alert the pharmacist and label the client's chart appropriately.

Category: Basic Physical Care A physician has ordered penicillin G potassium (Pfizerpen), I.V., for a client with a severe streptococcal infection. A nurse determines that the client may be allergic to penicillin. When considering best practice, what should the nurse's priority intervention be? a) Holding the penicillin G potassium and charting that it was held because the client is allergic b) Administering the penicillin G potassium and staying alert for any reaction c) Holding the penicillin G potassium and notifying the physician that the client may have an allergy to penicillin d) Administering the penicillin G potassium but notifying the pharmacist that the client is allergic

c) Holding the penicillin G potassium and notifying the physician that the client may have an allergy to penicillin Reason: The nurse should hold the penicillin G potassium, even if the client isn't sure he's allergic to penicillin, and notify the physician so he may order a different antibiotic. Many clients can't act as their own advocates; they rely on nurses to protect their rights. An allergy to penicillin G potassium is suspected, but not comfirmed. Administering penicillin G potassium could cause a life-threatening reaction. Administering the medication, then watching for a reaction or notifying the pharmacist that a reaction might occur, isn't best practice. If a client is allergic to penicillin, a nurse should alert the pharmacist and label the client's chart appropriately.

Category: Gastrointestinal Disorders A client who has been diagnosed with gastroesophageal reflux disease (GERD) complains of heartburn. To decrease the heartburn, the nurse should instruct the client to eliminate which of the following items from the diet? a) Lean beef. b) Air-popped popcorn. c) Hot chocolate. d) Raw vegetables.

c) Hot chocolate.

Category: Gastrointestinal Disorders A client who has a history of Crohn's disease is admitted to the hospital with fever, diarrhea, cramping, abdominal pain, and weight loss. The nurse should monitor the client for: a)Hyperalbuminemia. b)Thrombocytopenia. c)Hypokalemia. d)Hypercalcemia.

c) Hypokalemia. Reason: Hypokalemia is the most expected laboratory finding owing to the diarrhea. Hypoalbuminemia can also occur in Crohn's disease; however, the client's potassium level is of greater importance at this time because a low potassium level can cause cardiac arrest. Anemia is an expected development, but thrombocytopenia is not. Calcium levels are not affected.

Category: Foundations of Psychiatric Nursing The nursing staff has finished restraining a client. In addition to determining whether anyone was injured, the staff is mandated to evaluate the incident to obtain which of the following ultimate outcomes? a) Coordinate documentation of the incident. b) Resolve negative feelings and attitudes. c) Improve the use of restraint procedures. d) Calm down before returning to the other clients.

c) Improve the use of restraint procedures. Reason: Although coordinating documentation, resolving negative feelings, and calming down are goals of debriefing after a restraint, the ultimate outcome is to improve restraint procedures.

Category: The Nursing Process During rounds, a nurse finds that a client with hemiplegia has fallen from the bed because the nursing assistant failed to raise the side rails after giving a back massage. The nurse assists the client to the bed and assesses for injury. As per agency policies, the nurse fills out an incident report. Which of the following activities should the nurse perform after finishing the incident report? a) Attach a copy to the client's records. b) Highlight the mistake in the client's records. c) Include the time and date of the incident. d) Mention the name of the nursing assistant in the client records.

c) Include the time and date of the incident. Reason: The nurse should include the date and time of the incident in the incident report, the events leading up to it, the client's response, and a full nursing assessment. To prevent legal issues, the nurse should not attach the copy of the incident report to the client's records. Also to prevent litigation, the mistake should not be highlighted in the client's records. As the client report is a legal document, it should not contain the name of the nursing assistant.

Category: Neurosensory Disorders When obtaining the vital signs of a client with multiple traumatic injuries, a nurse detects bradycardia, bradypnea, and systolic hypertension. The nurse must notify the physician immediately because these findings may reflect which complication? a) Shock b) Encephalitis c) Increased intracranial pressure (ICP) d) Status epilepticus

c) Increased intracranial pressure (ICP) Reason: When ICP increases, Cushing's triad may develop, which involves decreased heart and respiratory rates and increased systolic blood pressure. Shock typically causes tachycardia, tachypnea, and hypotension. In encephalitis, the temperature rises and the heart and respiratory rates may increase from the effects of fever on the metabolic rate. (If the client doesn't maintain adequate hydration, hypotension may occur.) Status epilepticus causes unceasing seizures, not changes in vital signs.

Category: Neurosensory Disorders A client is receiving an I.V. infusion of mannitol (Osmitrol) after undergoing intracranial surgery to remove a brain tumor. To determine whether this drug is producing its therapeutic effect, the nurse should consider which finding most significant? a) Decreased level of consciousness (LOC) b) Elevated blood pressure c) Increased urine output d) Decreased heart rate

c) Increased urine output

Category: Neurosensory Disorders A client is receiving an I.V. infusion of mannitol (Osmitrol) after undergoing intracranial surgery to remove a brain tumor. To determine whether this drug is producing its therapeutic effect, the nurse should consider which finding most significant? a)Decreased level of consciousness (LOC) b)Elevated blood pressure c)Increased urine output d)Decreased heart rate

c) Increased urine output Reason: The therapeutic effect of mannitol is diuresis, which is confirmed by an increased urine output. A decreased LOC and elevated blood pressure may indicate lack of therapeutic effectiveness. A decreased heart rate doesn't indicate that mannitol is effective.

Category: School-age Child According to Erikson's psychosocial theory of development, an 8-year-old child would be in which stage? a) Trust versus mistrust b) Initiative versus guilt c) Industry versus inferiority d) Identity versus role confusion

c) Industry versus inferiority Reason: In middle childhood, the 6- to 12-year-old child is mastering the task of industry versus inferiority. The trust versus mistrust task is in infancy (birth to 1 year). In early childhood, the 1- to 3-year-old child is in the stage of initiative versus guilt. Identity versus role confusion occurs during adolescence.

Category: The Nursing Process The nurse is caring for a client with asthma. The nurse should conduct a focused assessment to detect which of the following? a) Increased forced expiratory volume. b) Normal breath sounds. c) Inspiratory and expiratory wheezing. d) Morning headaches.

c) Inspiratory and expiratory wheezing. Explanation: The hallmark signs of asthma are chest tightness, audible wheezing, and coughing. Inspiratory and expiratory wheezing is the result of bronchoconstriction. Even between exacerbations there may be some soft wheezing, so a finding of normal breath sounds would be expected in the absence of asthma. The expected finding is decreased forced expiratory volume (forced expiratory flow [FEF] is the flow [or speed] of air coming out of the lung during the middle portion of a forced expiration) due to bronchial constriction. Morning headaches are found in more advanced cases of COPD and signal nocturnal hypercapnia or hypoxemia.

Category: Antepartum Period Which medication is considered safe during pregnancy? a) Aspirin b) Magnesium hydroxide c) Insulin d) Oral antidiabetic agents

c) Insulin Reason: Insulin is a required hormone for any client with diabetes mellitus, including the pregnant client. Aspirin, magnesium hydroxide, and oral antidiabetic agents aren't recommended for use during pregnancy because these agents may cause fetal harm.

Category: Substance Abuse, Eating Disorders, Impulse Control Disorders Which of the following is a priority during the first 24 hours of hospitalization for a comatose client with suspected drug overdose? a)Educate regarding drug abuse. b)Minimize pain. c)Maintain intact skin. d)Increase caloric intake.

c) Maintain intact skin. Reason: Maintaining intact skin is a priority for the unconscious client. Unconscious clients need to be turned every hour to prevent complications of immobility, which include pressure ulcers and stasis pneumonia. The unconscious client cannot be educated at this time. Pain is not a concern. During the first 24 hours, the unconscious client will mostly likely be on nothing-by-mouth status.

Category: The Nursing Process An 18-year-old high school senior wishes to obtain birth control through her parents' insurance but does not want the information disclosed. The nurse tells the client that under the Health Information Portability and Accountability Act (HIPAA) parents: a) Have the right to review a minor's medical records until high school graduation. b) Have the right to review a minor's medical record if they are responsible for the payment. c) May not view the medical record, but may learn of the visit through the insurance bill. d) May not view the minor's medical record or the insurance bill.

c) May not view the medical record, but may learn of the visit through the insurance bill. Reason: Under HIPAA, 18-year-olds have the right to medical privacy and their medical records may not be disclosed to their parents without their permission. However, the adolescent must be made aware of the fact that information is sent to third party payers for the purpose of reimbursement. Those payers send the primary insurer, in this case the parent, a statement of benefits. HIPAA protects the right to medical privacy of all 18-year-olds regardless of their educational status. Even if parents are responsible for payment, they may not view the patient's chart without the consent of the adolescent.

Category: The Nursing Process An 18-year-old high school senior wishes to obtain birth control through her parents' insurance but does not want the information disclosed. The nurse tells the client that under the Health Information Portability and Accountability Act (HIPAA) parents: a) Have the right to review a minor's medical records until high school graduation. b) Have the right to review a minor's medical record if they are responsible for the payment. c) May not view the medical record, but may learn of the visit through the insurance bill. d) May not view the minor's medical record or the insurance bill.

c) May not view the medical record, but may learn of the visit through the insurance bill. Reason: Under HIPAA, 18-year-olds have the right to medical privacy and their medical records may not be disclosed to their parents without their permission. However, the adolescent must be made aware of the fact that information is sent to third party payers for the purpose of reimbursement. Those payers send the primary insurer, in this case the parent, a statement of benefits. HIPAA protects the right to medical privacy of all 18-year-olds regardless of their educational status. Even if parents are responsible for payment, they may not view the patient's chart without the consent of the adolescent.

Category: Adolescent A nurse is performing a psychosocial assessment on a 14-year-old adolescent. Which emotional response is typical during early adolescence? a) Frequent anger b) Cooperativeness c) Moodiness d) Combativeness

c) Moodiness

Category: Adolescent A nurse is performing a psychosocial assessment on a 14-year-old adolescent. Which emotional response is typical during early adolescence? a)Frequent anger b)Cooperativeness c)Moodiness d)Combativeness

c) Moodiness Reason: Moodiness may occur often during early adolescence. Frequent anger and combativeness are more typical of middle adolescence. Cooperativeness typically occurs during late adolescence.

Category: Musculoskeletal Disorders A client has been diagnosed with degenerative joint disease (osteoarthritis) of the left hip. Which of the following factors in the client's history would most likely increase the joint symptoms of osteoarthritis? a) A long history of smoking. b) Excessive alcohol use. c) Obesity. d) Emotional stress.

c) Obesity.

Category: Musculoskeletal Disorders A client has been diagnosed with degenerative joint disease (osteoarthritis) of the left hip. Which of the following factors in the client's history would most likely increase the joint symptoms of osteoarthritis? a)A long history of smoking. b)Excessive alcohol use. c)Obesity. d)Emotional stress.

c) Obesity. Reason: Osteoarthritis most commonly results from "wear and tear"---excessive and prolonged mechanical stress on the joints. Increased weight increases stress on weight-bearing joints. Therefore, an obese client with osteoarthritis should be encouraged to lose weight. Smoking does not cause osteoarthritis. Excessive alcohol use does not cause osteoarthritis. Emotional stress does not cause osteoarthritis.

Category: Antepartum Period A client has an episiotomy to widen her birth canal. Birth extends the incision into the anal sphincter. This complication is called: a) a first-degree laceration. b) a second-degree laceration. c) a third-degree laceration. d) a fourth-degree laceration.

c) a third-degree laceration. Reason: birth may extend an episiotomy incision to the anal sphincter (a third degree laceration) or the anal canal (a fourth degree laceration). A first degree laceration involves the fourchette, perineal skin, and vaginal mucous membranes. A second degree laceration extends to the fasciae and muscle of the perineal body.

Category: Musculoskeletal Disorders The nurse is evaluating the pin insertion site of a client's skeletal traction. Which of the following indicate a complication? a) Presence of crusts around the pin insertion site. b) Serous drainage on the dressing. c) Pin moves slightly at insertion site. d) Client does not feel pain at insertion site.

c) Pin moves slightly at insertion site. Explanation: Skeletal pins should not be loose and able to move. Any pin loosening should be reported immediately. Slight serous drainage is normal and may crust around the insertion site or be present on the dressing. The pin insertion site should be cleaned with aseptic technique according to facility policy. Pin insertion sites are typically not painful; pain may be indicative of an infection and should be reported.

Category: Musculoskeletal Disorders The nurse is evaluating the pin insertion site of a client's skeletal traction. Which of the following indicate a complication? a) Presence of crusts around the pin insertion site. b) Serous drainage on the dressing. c) Pin moves slightly at insertion site. d) Client does not feel pain at insertion site

c) Pin moves slightly at insertion site. Reason: Skeletal pins should not be loose and able to move. Any pin loosening should be reported immediately. Slight serous drainage is normal and may crust around the insertion site or be present on the dressing. The pin insertion site should be cleaned with aseptic technique according to facility policy. Pin insertion sites are typically not painful; pain may be indicative of an infection and should be reported.

Category: School-age Child The nurse should instruct the family of a child with newly diagnosed hyperthyroidism to: a)Keep their home warmer than usual.b)Encourage plenty of outdoor activities.c)Promote interactions with one friend instead of groups.d)Limit bathing to prevent skin irritation.

c) Promote interactions with one friend instead of groups. Reason: Children with hyperthyroidism experience emotional labiality that may strain interpersonal relationships. Focusing on one friend is easier than adapting to group dynamics until the child's condition improves. Because of their high metabolic rate, children with hyperthyroidism complain of being too warm. Bright sunshine may be irritating because of disease-related ophthalmopathy. Sweating is common and bathing should be encouraged.

Category: Infant During assessment of a small infant admitted with a diagnosis of meningitis, the infant becomes less responsive to stimuli and exhibits bradycardia, slight hypertension, irregular respirations, and a temperature of 103.2° F (39.6° C). The infant's fontanel is more tense than at the last assessment. What should the nurse do first? a) Ask another nurse to verify the findings. b) Notify the primary care provider of the findings. c) Raise the head of the bed. d) Administer an antipyretic.

c) Raise the head of the bed. Reason: Signs such as a decrease in the level of consciousness, bradycardia, hypertension, irregular respirations, and a tense fontanel strongly suggest increased intracranial pressure. The first action should be to attempt to lower the pressure by raising the head of the bed, which should improve venous return and decrease the pressure. Asking another nurse to verify the findings is unnecessary because temperature, pulse, and respirations are fairly objective data and not subject to interpretation. Additionally, asking for verification would waste valuable time. After elevating the infant's head by raising the bed, the nurse can notify the primary care provider and administer the antipyretic.

Category: Antepartum Period An anxious young adult is brought to the interviewing room of a crisis shelter, sobbing and saying that she thinks she is pregnant but does not know what to do. Which of the following nursing interventions is most appropriate at this time? a) Ask the client about the type of things that she had thought of doing. b) Give the client some ideas about what to expect to happen next. c) Recommend a pregnancy test after acknowledging the client's distress. d) Question the client about her feelings and possible parental reactions.

c) Recommend a pregnancy test after acknowledging the client's distress. Reason: Before any interventions can occur, knowing whether the client is pregnant is crucial in formulating a plan of care. Asking the client about what things she had thought about doing , giving the client some ideas about what to expect next, and questioning the client about her feelings and possible parental reactions would be appropriate after it is determined that the client is pregnant.

Category: Oncologic Disorders A client received chemotherapy 24 hours ago. Which precautions are necessary when caring for the client? a)Wear sterile gloves. b)Place incontinence pads in the regular trash container. c)Wear personal protective equipment when handling blood, body fluids, and feces. d)Provide a urinal or bedpan to decrease the likelihood of soiling linens.

c) Wear personal protective equipment when handling blood, body fluids, and feces. Reason: Chemotherapy drugs are present in the waste and body fluids of clients for 48 hours after administration. The nurse should wear personal protective equipment when handling blood, body fluids, or feces. Gloves offer minimal protection against exposure. The nurse should wear a face shield, gown, and gloves when exposure to blood or body fluid is likely. Placing incontinence pads in the regular trash container and providing a urinal or bedpan don't protect the nurse caring for the client.

Category: Respiratory Disorders Which of the following alert the nurse to possible internal bleeding in a client who has undergone pulmonary lobectomy 2 days ago? a) Increased blood pressure and decreased pulse and respiratory rates. b) Sanguineous drainage from the chest tube at a rate of 50 ml/hour during the past 3 hours. c) Restlessness and shortness of breath. d) Urine output of 180 ml during the past 3 hours.

c) Restlessness and shortness of breath. Reason: Restlessness indicates cerebral hypoxia due to decreased circulating volume. Shortness of breath occurs because blood collecting in the pleural space faster than suction can remove it prevents the lung from reexpanding. Increased blood pressure and decreased pulse and respiratory rates are classic late signs of increased intracranial pressure. Decreasing blood pressure and increasing pulse and respiratory rates occur with hypovolemic shock. Sanguineous drainage that changes to serosanguineous drainage at a rate less than 100 ml/hour is normal in the early postoperative period. Urine output of 180 ml over the past 3 hours indicates normal kidney perfusion.

Category: Respiratory Disorders Which of the following alert the nurse to possible internal bleeding in a client who has undergone pulmonary lobectomy 2 days ago? a)Increased blood pressure and decreased pulse and respiratory rates. b)Sanguineous drainage from the chest tube at a rate of 50 ml/hour during the past 3 hours. c)Restlessness and shortness of breath. d)Urine output of 180 ml during the past 3 hours.

c) Restlessness and shortness of breath. Reason: Restlessness indicates cerebral hypoxia due to decreased circulating volume. Shortness of breath occurs because blood collecting in the pleural space faster than suction can remove it prevents the lung from reexpanding. Increased blood pressure and decreased pulse and respiratory rates are classic late signs of increased intracranial pressure. Decreasing blood pressure and increasing pulse and respiratory rates occur with hypovolemic shock. Sanguineous drainage that changes to serosanguineous drainage at a rate less than 100 ml/hour is normal in the early postoperative period. Urine output of 180 ml over the past 3 hours indicates normal kidney perfusion.

Category: Basic Physical Care A man of Chinese descent is admitted to the hospital with multiple injuries after a motor vehicle accident. His pain is not under control. The client states, "If I could be with my people, I could receive acupuncture for this pain." The nurse should understand that acupuncture in the Asian culture is based on the theory that it: a) Purges evil spirits. b) Promotes tranquility. c) Restores the balance of energy. d) Blocks nerve pathways to the brain.

c) Restores the balance of energy.

Category: Basic Physical Care The nurse walks into a client's room to administer the 9:00 a.m. medications and notices that the client is in an awkward position in bed. What is the nurse's first action? a)Ask the client his name. b)Check the client's name band. c)Straighten the client's pillow behind his back. d)Give the client his medications.

c) Straighten the client's pillow behind his back. Reason: The nurse should first help the client into a position of comfort even though the primary purpose for entering the room was to administer medication. After attending to the client's basic care needs, the nurse can proceed with the proper identification of the client, such as asking the client his name and checking his armband, so that the medication can be administered.

Category: Respiratory Disorders A nurse is conducting an initial assessment on a client with possible tuberculosis. Which assessment finding indicates a risk factor for tuberculosis? a) The client sees his physician for a check-up yearly. b) The client has never traveled outside of the country. c) The client had a liver transplant 2 years ago. d) The client works in a health care insurance office.

c) The client had a liver transplant 2 years ago. Reason: A history of immunocompromised status, such as that which occurs with liver transplantation, places the client at a higher risk for contracting tuberculosis. Other risk factors include inadequate health care, traveling to countries with high rates of tuberculosis (such as southeastern Asia, Africa, and Latin America), being a health care worker who performs procedures in which exposure to respiratory secretions is likely, and being institutionalized.

Category: Mood, Adjustment, and Dementia Disorders A client was hospitalized for 1 week with major depression with suicidal ideation. He is taking venlafaxine (Effexor), 75 mg three times a day, and is planning to return to work. The nurse asks the client if he is experiencing thoughts of self-harm. The client responds, "I hardly think about it anymore and wouldn't do anything to hurt myself." The nurse should make which judgment about the client? a) The client is decompensating and in need of being readmitted to the hospital. b) The client needs an adjustment or increase in his dose of antidepressant. c) The depression is improving and the suicidal ideation is lessening. d) The presence of suicidal ideation warrants a telephone call to the client's primary care provider.

c) The depression is improving and the suicidal ideation is lessening. Reason: The client's statements about being in control of his behavior and his or her plans to return to work indicate an improvement in depression and that suicidal ideation, although present, is decreasing. Nothing in his comments or behavior indicate he is decompensating. There is no evidence to support an increase or adjustment in the dose of Effexor or a call to the primary care provider. Typically, the cognitive components of depression are the last symptoms eliminated. For the client to be experiencing some suicidal ideation in the second week of psychopharmacologic treatment is not unusual.

Category: Mood, Adjustment, and Dementia Disorders A nurse is evaluating a client's electrocardiogram (ECG). Which ECG change can result from amitriptyline (Elavil) therapy? a) Presence of U waves b) Depressed ST segment c) Widening QT interval d) Prolonged PR interval

c) Widening QT interval

Category: Mood, Adjustment, and Dementia Disorders A client was hospitalized for 1 week with major depression with suicidal ideation. He is taking venlafaxine (Effexor), 75 mg three times a day, and is planning to return to work. The nurse asks the client if he is experiencing thoughts of self-harm. The client responds, "I hardly think about it anymore and wouldn't do anything to hurt myself." The nurse should make which judgment about the client? a)The client is decompensating and in need of being readmitted to the hospital. b)The client needs an adjustment or increase in his dose of antidepressant. c)The depression is improving and the suicidal ideation is lessening. d)The presence of suicidal ideation warrants a telephone call to the client's primary care provider.

c) The depression is improving and the suicidal ideation is lessening. Reason: The client's statements about being in control of his behavior and his or her plans to return to work indicate an improvement in depression and that suicidal ideation, although present, is decreasing. Nothing in his comments or behavior indicate he is decompensating. There is no evidence to support an increase or adjustment in the dose of Effexor or a call to the primary care provider. Typically, the cognitive components of depression are the last symptoms eliminated. For the client to be experiencing some suicidal ideation in the second week of psychopharmacologic treatment is not unusual.

Category: Gastrointestinal Disorders Before an incisional cholecystectomy is performed, the nurse instructs the client in the correct use of an incentive spirometer. Why is incentive spirometry essential after surgery in the upper abdominal area? a) The client will be maintained on bed rest for several days. b) Ambulation is restricted by the presence of drainage tubes. c) The operative incision is near the diaphragm. d) The presence of a nasogastric tube inhibits deep breathing.

c) The operative incision is near the diaphram Reason: The incisions made for upper abdominal surgeries, such as cholecystectomies, are near the diaphragm and make deep breathing painful. Incentive spirometry, which encourages deep breathing, is essential to prevent atelectasis after surgery. The client is not maintained on bed rest for several days. The client is encouraged to ambulate by the first postoperative day, even with drainage tubes in place. Nasogastric tubes do not inhibit deep breathing and coughing.

Category: Basic Psychosocial Needs Before preparing a client for surgery, the nurse assists in developing a teaching plan. What is the primary purpose of preoperative teaching? a) To determine whether the client is psychologically ready for surgery b) To express concerns to the client about the surgery c) To reduce the risk of postoperative complications d) To explain the risks associated with the surgery and obtain informed consent

c) To reduce the risk of postoperative complications RATIONALE: Preoperative teaching helps reduce the risk of postoperative complications by telling the client what to expect and providing a chance for him to practice, before surgery, any required postoperative activities, such as breathing and leg exercises. The physician - not the nurse - is responsible for determining the client's psychosocial readiness for surgery. It's inappropriate for the nurse to express personal concerns about surgery to a client. The physician should describe alternative treatments and explain the risks to the client when obtaining informed consent.

Category: Toddler A child with a poor nutritional status and weight loss is at risk for a negative nitrogen balance. To help diagnose this problem, the nurse anticipates that the physician will order which laboratory test? a)Total iron-binding capacity b)Hemoglobin (Hb) c)Total protein d)Sweat test

c) Total protein Reason: The nurse anticipates the physician will order a total protein test because negative nitrogen balance may result from inadequate protein intake. Measuring total iron-binding capacity and Hb levels would help detect iron deficiency anemia, not a negative nitrogen balance. The sweat test helps diagnose cystic fibrosis, not a negative nitrogen balance.

Category: Toddler A child with a poor nutritional status and weight loss is at risk for a negative nitrogen balance. To help diagnose this problem, the nurse anticipates that the physician will order which laboratory test? a) Total iron-binding capacity b) Hemoglobin (Hb) c) Total protein d) Sweat test

c) Total protein. Reason: The nurse anticipates the physician will order a total protein test because negative nitrogen balance may result from inadequate protein intake. Measuring total iron-binding capacity and Hb levels would help detect iron deficiency anemia, not a negative nitrogen balance. The sweat test helps diagnose cystic fibrosis, not a negative nitrogen balance.

Category: Gastrointestinal Disorders Which of the following interventions would be most appropriate for the nurse to recommend to a client to decrease discomfort from hemorrhoids? a) Decrease fiber in the diet. b) Take laxatives to promote bowel movements. c) Use warm sitz baths. d) Decrease physical activity.

c) Use warm sitz baths. Explanation: Use of warm sitz baths can help relieve the rectal discomfort of hemorrhoids.

Category: The Neonate A newborn admitted with pyloric stenosis is lethargic and has poor skin turgor. The primary care provider has ordered I.V. fluids of dextrose water with sodium and potassium. The baby's admission potassium level is 3.4Meq/ L. The nurse should: a) Notify the primary care provider. b) Administer the ordered fluids. c) Verify that the infant has urinated. d) Have the potassium level redrawn.

c) Verify that the infant has urinated. Explanation: Normal serum potassium levels are 3.5-4.5 Meq/L. Elevated potassium levels can cause life threatening cardiac arrhythmias. The nurse must verify that the client has the ability to clear potassium through urination before administering the drug. Infants with pyloric stenosis frequently have low potassium levels due to vomiting. A level of 3.4Meq/l is not unexpected and should be corrected with the ordered fluids. The lab value does not need to be redrawn as the findings are consistent with the infant's condition.

Category: Oncologic Disorders A client received chemotherapy 24 hours ago. Which precautions are necessary when caring for the client? a) Wear sterile gloves. b) Place incontinence pads in the regular trash container. c) Wear personal protective equipment when handling blood, body fluids, and feces. d) Provide a urinal or bedpan to decrease the likelihood of soiling linens.

c) Wear personal protective equipment when handling blood, body fluids, and feces.

Category: Antepartum Period A client in the triage area who is at 19 weeks' gestation states that she has not felt her baby move in the past week and no fetal heart tones are found. While evaluating this client, the nurse identifies her as being at the highest risk for developing which problem? a) Abruptio placentae. b) Placenta previa. c) Disseminated intravascular coagulation. d) Threatened abortion.

c) disseminated intravascular coagulation Explanation: A fetus that has died and is retained in utero places the mother at risk for disseminated intravascular coagulation (DIC) because the clotting factors within the maternal system are consumed when the nonviable fetus is retained. The longer the fetus is retained in utero, the greater the risk of DIC. This client has no risk factors, history, or signs and symptoms that put her at risk for either abruptio placentae, such as sharp pain and "woody," firm consistency of the abdomen (abruption). HELLP syndrome is a complication of preeclampsia that does not occur before 20 weeks gestation unless a molar pregnancy is present. There is no evidence that she is threatening to abort as she has no cramping or vaginal bleeding.

Category: The Nursing Process When developing a care plan for a client with a do-not-resuscitate (DNR) order, a nurse should: a)withhold food and fluids. b)discontinue pain medications. c)ensure access to spiritual care providers upon the client's request. d)always make the DNR client the last in prioritization of clients.

c) ensure access to spiritual care providers upon the client's request. Reason: Ensuring access to spiritual care, if requested by the client, is an appropriate nursing action. A nurse should continue to administer appropriate doses of pain medication as needed to promote the client's comfort. A health care provider may not withhold food and fluids unless the client has a living will that specifies this action. A DNR order does not mean that the client does not require nursing care.

Category: Genitourinary Disorders A client with chronic renal failure (CRF) has developed faulty red blood cell (RBC) production. The nurse should monitor this client for: a) nausea and vomiting. b) dyspnea and cyanosis. c) fatigue and weakness. d) thrush and circumoral pallor

c) fatigue and weakness. Reason: RBCs carry oxygen throughout the body. Decreased RBC production diminishes cellular oxygen, leading to fatigue and weakness. Nausea and vomiting may occur in CRF but don't result from faulty RBC production. Dyspnea and cyanosis are associated with fluid excess, not CRF. Thrush, which signals fungal infection, and circumoral pallor, which reflects decreased oxygenation, aren't signs of CRF.

Category: The Neonate The neonate of a client with type 1 diabetes is at high risk for hypoglycemia. An initial sign the nurse should recognize as indicating hypoglycemia in a neonate is: a) peripheral acrocyanosis. b) bradycardia. c) lethargy. d) jaundice.

c) lethargy.

Category: Respiratory Disorders A client with cystic fibrosis develops pneumonia. To decrease the viscosity of respiratory secretions, the physician orders acetylcysteine (Mucomyst). Before administering the first dose, the nurse checks the client's history for asthma. Acetylcysteine must be used cautiously in a client with asthma because it: a) is a respiratory depressant. b) is a respiratory stimulant. c) may induce bronchospasm. d) inhibits the cough reflex.

c) may induce bronchospasm. Reason: Acetylcysteine must be used cautiously in a client with asthma because it may induce bronchospasm. The drug isn't a respiratory depressant or stimulant. It's a mucolytic agent that decreases the viscosity of respiratory secretions by altering the molecular composition of mucus. Acetylcysteine doesn't inhibit the cough reflex.

Category: Mood, Adjustment, and Dementia Disorders A client with major depression sleeps 18 to 20 hours per day, shows no interest in activities he previously enjoyed and reports a 17-lb (7.7-kg) weight loss over the past month. Because this is the client's first hospitalization, the physician is most likely to order: a) phenelzine (Nardil). b) thiothixene (Navane). c) nortriptyline (Pamelor). d) trifluoperazine (Stelazine).

c) nortriptyline (Pamelor).

Category: Mood, Adjustment, and Dementia Disorders A client with major depression sleeps 18 to 20 hours per day, shows no interest in activities he previously enjoyed and reports a 17-lb (7.7-kg) weight loss over the past month. Because this is the client's first hospitalization, the physician is most likely to order: a) phenelzine (Nardil). b) thiothixene (Navane). c) nortriptyline (Pamelor). d) trifluoperazine (Stelazine)

c) nortriptyline (Pamelor). Reason: Nortriptyline, a tricyclic antidepressant, is used in first-time drug therapy because it causes few anticholinergic and sedative adverse effects. Phenelzine isn't ordered initially because it may cause many adverse effects and necessitates dietary restrictions. Thiothixene and trifluoperazine are antipsychotic agents and, therefore, inappropriate for clients with uncomplicated depression.

Category: Integumentary Disorders Following a small-bowel resection, a client develops fever and anemia. The surface surrounding the surgical wound is warm to the touch and necrotizing fasciitis is suspected. Another manifestation that suggests necrotizing fasciitis is: a) erythema. b) leukocytosis. c) pressurelike pain. d) swelling.

c) pressurelike pain. Explanation: Severe pressurelike pain out of proportion to visible signs distinguishes necrotizing fasciitis from cellulitis. Erythema, leukocytosis, and swelling are present in both cellulitis and necrotizing fasciitis.

Category: Integumentary Disorders Following a small-bowel resection, a client develops fever and anemia. The surface surrounding the surgical wound is warm to the touch and necrotizing fasciitis is suspected. Another manifestation that suggests necrotizing fasciitis is: a) erythema. b) leukocytosis. c) pressurelike pain. d) swelling.

c) pressurelike pain. Reason: Severe pressurelike pain out of proportion to visible signs distinguishes necrotizing fasciitis from cellulitis. Erythema, leukocytosis, and swelling are present in both cellulitis and necrotizing fasciitis.

Category: Mood, Adjustment, and Dementia Disorders A 40-year-old executive who was unexpectedly laid off from work 2 days earlier complains of fatigue and an inability to cope. He admits drinking excessively over the previous 48 hours. This behavior is an example of: a) alcoholism. b) a manic episode. c) situational crisis. d) depression.

c) situational crisis. Reason: A situational crisis results from a specific event in the life of a person who is overwhelmed by the situation and reacts emotionally. Fatigue, insomnia, and inability to make decisions are common signs and symptoms. The situational crisis may precipitate behavior that causes a crisis (alcohol or drug abuse). There isn't enough information to label this client an alcoholic. A manic episode is characterized by euphoria and labile affect. Symptoms of depression are usually present for 2 or more weeks.

Category: Oncologic Disorders A young man with early-stage testicular cancer is scheduled for a unilateral orchiectomy. The client confides to the nurse that he is concerned about what effects the surgery will have on his sexual performance. Which of the following responses by the nurse provides accurate information about sexual performance after an orchiectomy? a) "Most impotence resolves in a couple of months." b) "You could have early ejaculation with this type of surgery." c) "We will refer you to a sex therapist because you will probably notice erectile dysfunction." d) "Because your surgery does not involve other organs or tissues, you'll likely not notice much change in your sexual performance."

d) "Because your surgery does not involve other organs or tissues, you'll likely not notice much change in your sexual performance."

Category: Anxiety Disorders A 16-year-old academically gifted boy is about to graduate from high school early, because he has completed all courses needed to earn a diploma. Within the last 3 months, he has experienced panic attacks that have forced him to leave classes early and occasionally miss a day of school. He is concerned that these attacks may hinder his ability to pursue a college degree. What would be the best response by the school nurse who has been helping him deal with his panic attacks? a) "It is natural to be worried about going into a new environment. I am sure with your abilities you will do well once you get settled." b) "You are putting too much pressure on yourself. You just need to relax more and things will be alright." c) "It might be best for you to postpone going to college. You need to get these panic attacks controlled first." d) "It sounds like you have a real concern about transitioning to college. I can refer you to a health care provider for assessment and treatment."

d) "It sounds like you have a real concern about transitioning to college. I can refer you to a health care provider for assessment and treatment."

Category: Neurosensory Disorders A nurse is caring for a client diagnosed with a cerebral aneurysm who reports a severe headache. Which action should the nurse perform? a) Sit with the client for a few minutes. b) Administer an analgesic. c) Inform the nurse manager. d) Call the physician immediately.

d) Call the physician immediately.

Category: The Neonate During the first feeding, the nurse observes that the neonate becomes cyanotic after gagging on mucus. Which of the following should the nurse do first? a) Start mouth-to-mouth resuscitation. b) Contact the neonatal resuscitation team. c) Raise the neonate's head and pat the back gently. d) Clear the neonate's airway with suction or gravity

d) Clear the neonate's airway with suction or gravity

Category: Anxiety Disorders A 16-year-old academically gifted boy is about to graduate from high school early, because he has completed all courses needed to earn a diploma. Within the last 3 months, he has experienced panic attacks that have forced him to leave classes early and occasionally miss a day of school. He is concerned that these attacks may hinder his ability to pursue a college degree. What would be the best response by the school nurse who has been helping him deal with his panic attacks? a)"It is natural to be worried about going into a new environment. I am sure with your abilities you will do well once you get settled." b)"You are putting too much pressure on yourself. You just need to relax more and things will be alright." c)"It might be best for you to postpone going to college. You need to get these panic attacks controlled first." d)"It sounds like you have a real concern about transitioning to college. I can refer you to a health care provider for assessment and treatment."

d) "It sounds like you have a real concern about transitioning to college. I can refer you to a health care provider for assessment and treatment." Reason: The client's concerns are real and serious enough to warrant assessment by a physician rather than being dismissed as trivial. Though he is very intelligent, his intelligence cannot overcome his anxiety. In fact, his anxiety is likely to interfere with his ability to perform in college if no assessment and treatment are received. Just postponing college is likely to increase rather than lower the client's anxiety, because it does not address the panic he is experiencing.

Category: Antepartum Period A 16-year-old primigravida at 36 weeks' gestation who has had no prenatal care experienced a seizure at work and is being transported to the hospital by ambulance. Which of the following should the nurse do upon the client's arrival? a) Position the client in a supine position. b) Auscultate breath sounds every 4 hours. c) Monitor the vital signs every 4 hours. d) Admit the client to a quiet, darkened room.

d) Admit the client to a quiet, darkened room Reason: Because of her age and report of a seizure, the client is probably experiencing eclampsia, a condition in which convulsions occur in the absence of any underlying cause. Although the actual cause is unknown, adolescents and women older than 35 years are at higher risk. The client's environment should be kept as free of stimuli as possible. Thus, the nurse should admit the client to a quiet, darkened room. Clients experiencing eclampsia should be kept on the left side to promote placental perfusion. In some cases, edema of the lungs develops after seizures and is a sign of cardiovascular failure. Because the client is at risk for pulmonary edema, breath sounds should be monitored every 2 hours. Vital signs should be monitored frequently, at least every hour.

Category: The Neonate Nurses teach infant care and safety classes to assist parents in appropriately preparing to take their neonates home. Which statement about automobile restraints for infants is correct? a) An infant should ride in a front-facing car seat until he weighs 20 lb (9.1 kg) and is 1 year old. b) An infant should ride in a rear-facing car seat until he weighs 25 lb (11.3 kg) or is 1 year old. c) An infant should ride in a front-facing car seat until he weighs 30 lb (13.6 kg) or is 2 years old. d) An infant should ride in a rear-facing car seat until he weighs 20 lb and is 1 year old.

d) An infant should ride in a rear-facing car seat until he weighs 20 lb and is 1 year old. Reason: Until the infant weighs 20 lb and is 1 year old, he should ride in a rear-facing car seat.

Category: Psychotic Disorders The client is taking risperidone (Risperdal) to treat the positive and negative symptoms of schizophrenia. Improvement of which of the following negative symptoms indicate the drug is effective? a) Abnormal thought form. b) Hallucinations and delusions. c) Bizarre behavior. d) Asocial behavior and anergia.

d) Asocial behavior and anergia. Reason: Asocial behavior, anergia, alogia, and affective flattening are some of the negative symptoms of schizophrenia that may improve with risperidone therapy. Abnormal thought form is a positive symptom of schizophrenia. Hallucinations and delusions are positive symptoms of schizophrenia. Bizarre behavior is a positive symptom of schizophrenia.

Category: The Nursing Process The nurse is assigning tasks to unlicensed assistive personnel (UAP) for a client with an abdominal hysterectomy on the first postoperative day. Which of the following can NOT be delegated to the UAP? a) Taking vital signs. b) Recording intake and output. c) Giving perineal care. d) Assessing the incision site.

d) Assessing the incision site. Reason: The registered nurse is responsible for monitoring the surgical site for condition ofthe dressing, status of the incision, and signs and symptoms of complications. Unlicensedassistive personnel who have been trained to report abnormalities to the registered nursesupervising the care may take vital signs, record intake and output, and give perineal care.

Category: The Neonate During the first feeding, the nurse observes that the neonate becomes cyanotic after gagging on mucus. Which of the following should the nurse do first? a)Start mouth-to-mouth resuscitation. b)Contact the neonatal resuscitation team. c)Raise the neonate's head and pat the back gently. d)Clear the neonate's airway with suction or gravity.

d) Clear the neonate's airway with suction or gravity. Reason: If a neonate gags on mucus and becomes cyanotic during the first feeding, the airway is most likely closed. The nurse should clear the airway by gravity (by lowering the infant's head) or suction. Starting mouth-to-mouth resuscitation is not indicated unless the neonate remains cyanotic and lowering his head or suctioning doesn't clear his airway. Contacting the neonatal resuscitation team is not warranted unless the infant remains cyanotic even after measures to clear the airway. Raising the neonate's head and patting the back are not appropriate actions for removing mucus. Doing so allows the mucus to remain lodged causing further breathing difficulties.

Category: Toddler After teaching the parents of an 18-month-old who was treated for a foreign body obstruction about the three cardinal signs indicative of choking, the nurse determines that the teaching has been successful when the parents state that a child is choking when he or she cannot speak, turns blue, and does which of the following? a) Vomits. b) Gasps. c) Gags. d) Collapses.

d) Collapses. Reason: The three cardinal signs indicating that a child is truly choking and requires immediate life-saving interventions include inability to speak, blue color (cyanosis), and collapse. Vomiting does not occur while a child is unable to breathe. Once the object is dislodged, however, vomiting may occur. Gasping, a sudden intake of air, indicates that the child is still able to inhale. When a child is choking, air is not being exchanged, so gagging will not occur.

Category: Medication and I.V. Administration A 56-year-old client is receiving chemotherapy that has the potential to cause pulmonary toxicity. Which of the following symptoms indicates a toxic response to the chemotherapy? a) Decrease in appetite. b) Drowsiness. c) Spasms of the diaphragm. d) Cough and shortness of breath.

d) Cough and shortness of breath. Reason: Cough and shortness of breath are significant symptoms because they may indicate decreasing pulmonary function secondary to drug toxicity. Decrease in appetite, difficulty in thinking clearly, and spasms of the diaphragm may occur as a result of chemotherapy; however, they are not indicative of pulmonary toxicity.

Category: The Nursing Process Which of the following should be included in the plan of care for a client with a surgical wound that requires a wet-to-dry dressing? a) Place a dry dressing in the wound. b) Use Burrow's solution to wet the dressing. c) Pack the wet dressing tightly into the wound. d) Cover the wet packing with a dry sterile dressing.

d) Cover the wet packing with a dry sterile dressing. Reason: A wet-to-dry dressing should be able to dry out between dressing changes. Thus, the dressing should be moist, not dry, when applied. As the moist dressing dries, the wound will be debrided of necrotic tissue, exudate, and so forth. Normal saline is most commonly used to moisten the sponge; Burrow's solution will irritate the wound. The sponge should not be packed into the wound tightly because the circulation to the site could be impaired. The moist sponge should be placed so that all surfaces of the wound are in contact with the dressing. Then the sponge is covered and protected by a dry sterile dressing to prevent contamination from the external environment.

Category: Endocrine and Metabolic Disorders A client is diagnosed with diabetes mellitus. Which assessment finding best supports a nursing diagnosis of Ineffective coping related to diabetes mellitus? a) Recent weight gain of 20 lb (9.1 kg) b) Failure to monitor blood glucose levels c) Skipping insulin doses during illness d) Crying whenever diabetes is mentioned

d) Crying whenever diabetes is mentioned

Category: Endocrine and Metabolic Disorders A client is diagnosed with diabetes mellitus. Which assessment finding best supports a nursing diagnosis of Ineffective coping related to diabetes mellitus? a) Recent weight gain of 20 lb (9.1 kg) b) Failure to monitor blood glucose levels c) Skipping insulin doses during illness d) Crying whenever diabetes is mentioned

d) Crying whenever diabetes is mentioned Reason: A client who cries whenever diabetes is mentioned is demonstrating ineffective coping. A recent weight gain and failure to monitor blood glucose levels would support a nursing diagnosis of Noncompliance: Failure to adhere to therapeutic regimen. Skipping insulin doses during illness would support a nursing diagnosis of Deficient knowledge related to treatment of diabetes mellitus.

Category: Endocrine and Metabolic Disorders Which finding best indicates that a nursing assistant has an understanding of blood glucose meter use? a) Verbalizing an understanding of blood glucose meter use b) Documenting a normal blood glucose level c) Providing documentation of previous certification d) Demonstrating correct technique

d) Demonstrating correct technique RATIONALES: The best way to validate blood glucose meter use is to allow the nursing assistant to demonstrate correct technique. Verbalizing understanding doesn't demonstrate that the nursing assistant knows proper technique. Options 2 and 3 don't demonstrate blood glucose meter use.

Category: Genitourinary Disorders The nurse is assessing the urine of a client who has had an ileal conduit and notes that the urine is yellow with a moderate amount of mucus. Based on these data, the nurse should? a) Change the appliance bag. b) Notify the physician. c) Obtain a urine specimen for culture. d) Encourage a high fluid intake.

d) Encourage a high fluid intake. Reason: Mucus is secreted by the intestinal segment used to create the conduit and is a normal occurrence. The client should be encouraged to maintain a large fluid intake to help flush the mucus out of the conduit. Because mucus in the urine is expected, it is not necessary to change the appliance bag or to notify the physician. The mucus is not an indication of an infection, so a urine culture is not necessary.

Category: Medication and I.V. Administration The nurse administers an intradermal injection to a client. Proper technique has been used if the injection site demonstrates which of the following? a) Minimal leaking. b) No swelling. c) Tissue pallor. d) Evidence of a bleb or wheal.

d) Evidence of a bleb or wheal. Reason: A properly administered intradermal injection shows evidence of a bleb or wheal at the injection site. There should be no leaking of medication from the bleb; it needs to be absorbed into the tissue. Lack of swelling at the injection site means that the injection was given too deeply. The presence of tissue pallor does not indicate that the injection was given correctly.

Category: Foundations of Psychiatric Nursing A client in an acute care setting tells the nurse, "I don't think I can face going home tomorrow." The nurse replies, "Do you want to talk more about it?" The nurse is using which technique? a)Presenting reality b)Making observations c)Restating d)Exploring

d) Exploring Explanation: The nurse is using the technique of exploring because she's willing to delve further into the client's concern. She isn't presenting reality or making observations or simply restating. The nurse is encouraging the client to explore his feelings.

Category: Foundations of Psychiatric Nursing A client in an acute care setting tells the nurse, "I don't think I can face going home tomorrow." The nurse replies, "Do you want to talk more about it?" The nurse is using which technique? a) Presenting reality b) Making observations c) Restating d) Exploring

d) Exploring RATIONALE: The nurse is using the technique of exploring because she's willing to delve further into the client's concern. She isn't presenting reality or making observations or simply restating. The nurse is encouraging the client to explore his feelings.

Category: Basic Physical Assessment Crackles heard on lung auscultation indicate which of the following? a) Cyanosis. b) Bronchospasm. c) Airway narrowing. d) Fluid-filled alveoli.

d) Fluid-filled alveoli. Reason: Crackles are auscultated over fluid-filled alveoli. Crackles heard on lung auscultation do not have to be associated with cyanosis. Bronchospasm and airway narrowing generally are associated with wheezing sounds.

Category: The Neonate While assessing a male neonate whose mother desires him to be circumcised, the nurse observes that the neonate's urinary meatus appears to be located on the ventral surface of the penis. The primary health care provider is notified because the nurse suspects which of the following? a) Phimosis. b) Hydrocele. c) Epispadias. d) Hypospadias.

d) Hypospadias. Reason: The condition in which the urinary meatus is located on the ventral surface of the penis, termed hypospadias, occurs in 1 of every 500 male infants. Circumcision is delayed until the condition is corrected surgically, usually between 6 and 12 months of age. Phimosis is an inability to retract the prepuce at an age when it should be retractable or by age 3 years. Phimosis may necessitate circumcision or surgical intervention. Hydrocele is a painless swelling of the scrotum that is common in neonates. It is not a contraindication for circumcision. Epispadias occurs when the urinary meatus is located on the dorsal surface of the penis. It is extremely rare and is commonly associated with bladder extrophy.

Category: Neurosensory Disorders Which of the following is an early symptom of glaucoma? a) Hazy vision. b) Loss of central vision. c) Blurred or "sooty" vision. d) Impaired peripheral vision.

d) Impaired peripheral vision. Reason: In glaucoma, peripheral vision is impaired long before central vision is impaired. Hazy, blurred, or distorted vision is consistent with a diagnosis of cataracts. Loss of central vision is consistent with senile macular degeneration but it occurs late in glaucoma. Blurred or "sooty" vision is consistent with a diagnosis of detached retina.

Category: Respiratory Disorders A client with a tracheostomy tube coughs and dislodges the tracheostomy tube. The nurse's first action should be to: a) Call for emergency assistance. b) Attempt reinsertion of tracheostomy tube. c) Position the client in semi-Fowler's position with the neck hyperextended. d) Insert the obturator into the stoma to reestablish the airway.

d) Insert the obturator into the stoma to reestablish the airway.

Category: Respiratory Disorders A client hospitalized for treatment of a pulmonary embolism develops respiratory alkalosis. Which clinical findings commonly accompany respiratory alkalosis? a) Nausea or vomiting b) Abdominal pain or diarrhea c) Hallucinations or tinnitus d) Light-headedness or paresthesia

d) Light-headedness or paresthesia Reason: The client with respiratory alkalosis may complain of light-headedness or paresthesia (numbness and tingling in the arms and legs). Nausea, vomiting, abdominal pain, and diarrhea may accompany respiratory acidosis. Hallucinations and tinnitus rarely are associated with respiratory alkalosis or any other acid-base imbalance.

Category: Infant An infant is hospitalized for treatment of inorganic failure to thrive. Which nursing action is most appropriate for this child? a) Encouraging the infant to hold a bottle b) Keeping the infant on bed rest to conserve energy c) Rotating caregivers to provide more stimulation d) Maintaining a consistent, structured environment

d) Maintaining a consistent, structured environment Reason: The nurse caring for an infant with inorganic failure to thrive should strive to maintain a consistent, structured environment because it reinforces a caring feeding environment. Encouraging the infant to hold a bottle would reinforce an uncaring feeding environment. The infant should receive social stimulation rather than be confined to bed rest. The number of caregivers should be minimized to promote consistency of care.

Category: The Nursing Process A 57-year-old Hispanic woman with breast cancer who does not speak English is admitted for a lumpectomy. Her daughter, who speaks English, accompanies her. In order to obtain admission information from the client, what should the nurse do? a) Ask the client's daughter to serve as an interpreter. b) Ask one of the Hispanic nursing assistants to serve as an interpreter. c) Use the limited Spanish she remembers from high school along with nonverbal communication. d) Obtain a trained medical interpreter.

d) Obtain a trained medical interpreter. Reason: a trained medical interpreter is requried to ensure safety, accuracy of history data and client's confidentiality. The medical interpreter knows the client's rights and is familiar with the client's culture.

Category: Foundations of Psychiatric Nursing A client was talking with her husband by telephone, and then she began swearing at him. The nurse interrupts the call and offers to talk with the client. She says, "I can't talk about that bastard right now. I just need to destroy something." Which of the following should the nurse do next? a) Tell her to write her feelings in her journal. b) Urge her to talk with the nurse now. c) Ask her to calm down or she will be restrained. d) Offer her a phone book to "destroy" while staying with her.

d) Offer her a phone book to "destroy" while staying with her. Reason: At this level of aggression, the client needs an appropriate physical outlet for the anger. She is beyond writing in a journal. Urging the client to talk to the nurse now or making threats, such as telling her that she will be restrained, is inappropriate and could lead to an escalation of her anger.

Category: Integumentary Disorders A nurse is performing a baseline assessment of a client's skin integrity. What is the priority assessment parameter? a) Family history of pressure ulcers b) Presence of pressure ulcers on the client c) Potential areas of pressure ulcer development d) Overall risk of developing pressure ulcers

d) Overall risk of developing pressure ulcers Reason: When assessing skin integrity, the overall risk potential of developing pressure ulcers takes priority. Overall risk encompasses existing pressure ulcers as well as potential areas for development of pressure ulcers. Family history isn't important when assessing skin integrity.

Category: Integumentary Disorders A nurse is performing a baseline assessment of a client's skin integrity. What is the priority assessment parameter? a)Family history of pressure ulcers b)Presence of pressure ulcers on the client c)Potential areas of pressure ulcer development d)Overall risk of developing pressure ulcers

d) Overall risk of developing pressure ulcers Reason: When assessing skin integrity, the overall risk potential of developing pressure ulcers takes priority. Overall risk encompasses existing pressure ulcers as well as potential areas for development of pressure ulcers. Family history isn't important when assessing skin integrity.

Category: Postpartum Period A client is experiencing an early postpartum hemorrhage. Which item in the client's care plan requires revision? a) Inserting an indwelling urinary catheter b) Fundal massage c) Administration of oxytocics d) Pad count

d) Pad count Reason: By the time the client is hemorrhaging, a pad count is no longer appropriate.

Category: Preschooler A child diagnosed with tetralogy of Fallot becomes upset, crying and thrashing around when a blood specimen is obtained. The child's color becomes blue and the respiratory rate increases to 44 breaths/minute. Which of the following actions should the nurse do first? a) Obtain an order for sedation for the child. b) Assess for an irregular heart rate and rhythm. c) Explain to the child that it will only hurt for a short time. d) Place the child in a knee-to-chest position.

d) Place the child in a knee-to-chest position. Reason: The child is experiencing a tet or hypoxic episode. Therefore the nurse should place the child in a knee-to-chest position. Flexing the legs reduces venous flow of blood from the lower extremities and reduces the volume of blood being shunted through the interventricular septal defect and the overriding aorta in the child with tetralogy of Fallot. As a result, the blood then entering the systemic circulation has a higher oxygen content, and dyspnea is reduced. Flexing the legs also increases vascular resistance and pressure in the left ventricle. An infant often assumes a knee-to-chest position in the crib, or the mother learns to put the infant over her shoulder while holding the child in a knee-to-chest position to relieve dyspnea. If this position is ineffective, then the child may need a sedative. Once the child is in the position, the nurse may assess for an irregular heart rate and rhythm. Explaining to the child that it will only hurt for a short time does nothing to alleviate the hypoxia.

Category: Mood, Adjustment, and Dementia Disorders A client diagnosed with a cognitive disorder is showing signs of confusion, short-term memory loss, and a short attention span. Which of the following therapy groups would be best suited for this client? a) Insight-oriented. b) Medication management. c) Problem solving. d) Reality-orientation.

d) Reality-orientation. Reason: Because the client has confusion, short-term memory loss, and a short attention span, a reality-orientation group is recommended to help the client maintain an optimal level of functioning, decrease isolation, and increase self-esteem. Focus is on the "here and now" and provides reality testing, structure, and social support. A client with a cognitive disorder is unlikely to benefit from an insight-oriented group, where the focus is on role relationships. Short-term memory loss and confusion interfere with the ability to learn about medication management. Short-term memory loss and confusion interfere with the ability to describe and solve problems.

Category: Preschooler A 4-year-old boy presents to the emergency department. His father tearfully reports that he was in the driveway and had his son on his shoulders when the child began to fall. The father grabbed him by the leg, swinging him toward the grass to avoid landing on the pavement. As the father swung his son, the child hit his head on the driveway and twisted his right leg. After a complete examination, it is determined that the child has a skull fracture and a spiral fracture of the femur. Which of the following actions should the nurse take? a) Restrict the father's visitation. b) Notify the police immediately. c) Refer the father for parenting classes. d) Record the father's story in the chart.

d) Record the father's story in the chart. Reason: The father's story is consistent with the injuries incurred by the child; therefore, the nurse should document the cause of injury. There is no need to restrict the father's visitation, because the injuries sustained by the child are consistent with the explanation given. The police need to be notified only if there is suspicion of child abuse. The injuries incurred by this child appear accidental. There is no need to refer the father for parenting classes. The father seems upset about the accident and will not likely repeat such reckless behavior. The nurse should educate the father, however, regarding child safety.

Category: Neurosensory Disorders A potential concern when caring for an older adult who has diminished hearing and vision is the client's: a) Feelings of disorientation. b) Cognitive impairment. c) Sensory overload. d) Social isolation.

d) Social isolation. Reason: Social isolation is a concern for an older adult who has diminished hearing and vision. Feeling disoriented may be related to cognitive problems rather than diminished hearing and vision. Diminished hearing and vision is related to the aging process and does not result in impairment of the older adult's thought processes. The client with impaired hearing and vision is unlikely to experience sensory overload.

Category: The Nursing Process A nurse is caring for a client with a diagnosis of Impaired gas exchange. Based upon this nursing diagnosis, which outcome is most appropriate? a) The client maintains a reduced cough effort to lessen fatigue. b) The client restricts fluid intake to prevent overhydration. c) The client reduces daily activities to a minimum. d) The client has normal breath sounds in all lung fields.

d) The client has normal breath sounds in all lung fields. Reason: If the interventions are effective, the client's breath sounds should return to normal. The client should be able to cough effectively and should be encouraged to increase activity, as tolerated. Fluids should help thin secretions, so fluid intake should be encouraged.

Category: The Nursing Process A nurse is caring for a client with a diagnosis of Impaired gas exchange. Based upon this nursing diagnosis, which outcome is most appropriate? a)The client maintains a reduced cough effort to lessen fatigue. b)The client restricts fluid intake to prevent overhydration. c)The client reduces daily activities to a minimum. d)The client has normal breath sounds in all lung fields.

d) The client has normal breath sounds in all lung fields. Reason: If the interventions are effective, the client's breath sounds should return to normal. The client should be able to cough effectively and should be encouraged to increase activity, as tolerated. Fluids should help thin secretions, so fluid intake should be encouraged.

Category: Foundations of Psychiatric Nursing Based on a client's history of violence toward others and her inability to cope with anger, which of the following should the nurse use as the most important indicator of goal achievement before discharge? a) Acknowledgment of her angry feelings. b) Ability to describe situations that provoke angry feelings. c) Development of a list of how she has handled her anger in the past. d) Verbalization of her feelings in an appropriate manner.

d) Verbalization of her feelings in an appropriate manner. Reason: Verbalizing feelings, especially feelings of anger, in an appropriate manner is an adaptive method of coping that reduces the chance that the client will act out these feelings toward others. The client's ability to verbalize her feelings indicates a change in behavior, a crucial indicator of goal achievement. Although acknowledging feelings of anger and describing situations that precipitate angry feelings are important in helping the client reach her goal, they are not appropriate indicators that she has changed her behavior. Asking the client to list how she has handled anger in the past is helpful if the nurse discusses coping methods with the client. However, based on this client's history, this would not be helpful because the nurse and client are already aware of the client's aggression toward others.

Category: Foundations of Psychiatric Nursing Based on a client's history of violence toward others and her inability to cope with anger, which of the following should the nurse use as the most important indicator of goal achievement before discharge? a)Acknowledgment of her angry feelings. b)Ability to describe situations that provoke angry feelings. c)Development of a list of how she has handled her anger in the past. d)Verbalization of her feelings in an appropriate manner.

d) Verbalization of her feelings in an appropriate manner. Reason: Verbalizing feelings, especially feelings of anger, in an appropriate manner is an adaptive method of coping that reduces the chance that the client will act out these feelings toward others. The client's ability to verbalize her feelings indicates a change in behavior, a crucial indicator of goal achievement. Although acknowledging feelings of anger and describing situations that precipitate angry feelings are important in helping the client reach her goal, they are not appropriate indicators that she has changed her behavior. Asking the client to list how she has handled anger in the past is helpful if the nurse discusses coping methods with the client. However, based on this client's history, this would not be helpful because the nurse and client are already aware of the client's aggression toward others.

Category: Psychotic Disorders A client has refused to take a shower since being admitted 4 days earlier. He tells a nurse, "There are poison crystals hidden in the showerhead. They'll kill me if I take a shower." Which nursing action is most appropriate? a) Dismantling the showerhead and showing the client that there is nothing in it b) Explaining that other clients are complaining about the client's body odor c) Asking a security officer to assist in giving the client a shower d) Accepting these fears and allowing the client to take a sponge bath

d) accepting these fears and allowing the client to take a sponge bath Reason: by acknowledging the client's fears, the nurse can arrange to meet the client's hygiene needs in another way. Because these fears are real to the client, providing a demonstration of reality by dismantling the shower head wouldn't be effective at this time.

Category: Postpartum Period On the second postpartum day a gravida 6, para 5 complains of intermittent abdominal cramping. The nurse should assess for: a) endometritis. b) postpartum hemorrhage. c) subinvolution. d) afterpains.

d) afterpains. Explanation: In a multiparous client, decreased uterine muscle tone causes alternating relaxation and contraction during uterine involution, which leads to afterpains. The client's symptoms don't suggest endometritis, hemorrhage, or subinvolution.

Category: Postpartum Period On the second postpartum day a gravida 6, para 5 complains of intermittent abdominal cramping. The nurse should assess for: a)endometritis. b)postpartum hemorrhage. c)subinvolution. d)afterpains.

d) afterpains. Reason: In a multiparous client, decreased uterine muscle tone causes alternating relaxation and contraction during uterine involution, which leads to afterpains. The client's symptoms don't suggest endometritis, hemorrhage, or subinvolution.

Category: Antepartum Period A nurse is providing care for a pregnant client in her second trimester. Glucose tolerance test results show a blood glucose level of 160 mg/dl. The nurse should anticipate that the client will need to: a) start using insulin. b) start taking an oral antidiabetic drug. c) monitor her urine for glucose. d) be taught about diet.

d) be taught about diet.

Category: Basic Physical Care A client with burns on his groin has developed blisters. As the client is bathing, a few blisters break. The best action for the nurse to take is to: a) remove the raised skin because the blister has already broken. b) wash the area with soap and water to disinfect it. c) apply a weakened alcohol solution to clean the area. d) clean the area with normal saline solution and cover it with a protective dressing.

d) clean the area with normal saline solution and cover it with a protective dressing. Reason: The nurse should clean the area with a mild solution such as normal saline, and then cover it with a protective dressing. Soap and water and alcohol are too harsh. The body's first line of defense broke when the blisters opened; removing the skin exposes a larger area to the risk of infection.

Category: Basic Physical Assessment When assessing an elderly client, the nurse expects to find various aging-related physiologic changes. These changes include: a) increased coronary artery blood flow. b) decreased posterior thoracic curve. c) decreased peripheral resistance. d) delayed gastric emptying.

d) delayed gastric emptying. Reason: Aging-related physiologic changes include delayed gastric emptying, decreased coronary artery blood flow, an increased posterior thoracic curve, and increased peripheral resistance.

Category: Basic Physical Assessment When assessing an elderly client, the nurse expects to find various aging-related physiologic changes. These changes include: a)increased coronary artery blood flow. b)decreased posterior thoracic curve. c)decreased peripheral resistance. d)delayed gastric emptying.

d) delayed gastric emptying. Reason: Aging-related physiologic changes include delayed gastric emptying, decreased coronary artery blood flow, an increased posterior thoracic curve, and increased peripheral resistance.

Category: Psychotic Disorders A client with chronic undifferentiated schizophrenia is admitted to the psychiatric unit of a local hospital. During the next several days, the client is seen laughing, yelling, and talking to himself. This behavior is characteristic of: a) delusion. b) looseness of association. c) illusion. d) hallucination.

d) hallucination. Explanation: Auditory hallucination, in which one hears voices when no external stimuli exist, is common in schizophrenic clients. Such behaviors as laughing, yelling, and talking to oneself suggest such a hallucination. Delusions, also common in schizophrenia, are false beliefs or ideas that arise without external stimuli. Clients with schizophrenia may exhibit looseness of association, a pattern of thinking and communicating in which ideas aren't clearly linked to one another. Illusion is a less severe perceptual disturbance in which the client misinterprets actual external stimuli. Illusions are rarely associated with schizophrenia.

Category: Psychotic Disorders A client with chronic undifferentiated schizophrenia is admitted to the psychiatric unit of a local hospital. During the next several days, the client is seen laughing, yelling, and talking to himself. This behavior is characteristic of: a)delusion. b)looseness of association. c)illusion. d)hallucination.

d) hallucination. Reason: Auditory hallucination, in which one hears voices when no external stimuli exist, is common in schizophrenic clients. Such behaviors as laughing, yelling, and talking to oneself suggest such a hallucination. Delusions, also common in schizophrenia, are false beliefs or ideas that arise without external stimuli. Clients with schizophrenia may exhibit looseness of association, a pattern of thinking and communicating in which ideas aren't clearly linked to one another. Illusion is a less severe perceptual disturbance in which the client misinterprets actual external stimuli. Illusions are rarely associated with schizophrenia.

Category: Foundations of Psychiatric Nursing A client diagnosed with pain disorder is talking with the nurse about fishing when he suddenly reverts to talking about the pain in his arm. Which of the following should the nurse do next? a) Allow the client to talk about his pain. b) Ask the client if he needs more pain medication. c) Get up and leave the client. d) Redirect the interaction back to fishing.

d) redirect the interaction back to fishing Reason: Doing so helps the pt talk about topics that are more therapeutic/beneficial to recovery. Allowing pt to talk about pain or asking if he needs additional pain med is not therapeutic because it reinforces the client's need for the symptom. Getting up and leaving the client is not appropriate unless the nurse has set limits previously by saying, "I will get up and leave if you continue to talk about your pain."


संबंधित स्टडी सेट्स

Hemodynamic Quiz 207/Unit4 (Exam 4)

View Set

CH. 9 Fundamentals of Nursing- PrepU Quiz

View Set

Human Growth and Development Study Set Test 1

View Set

Retirement Plans and Social Security

View Set

Business Law Semester 1 Study Guide

View Set

Laws and Protocols for Air Pollution

View Set